CH 37: Acute Coronary Syndrome Flashcards
A 58-year-old male with a history of hypertension and hyperlipidemia presents to the emergency department with crushing substernal chest pain radiating to his left arm. His ECG shows ST-segment elevations in leads V2-V4. The nurse recognizes that immediate intervention is needed.
Which of the following is the priority action?
A. Administer aspirin and clopidogrel.
B. Prepare for immediate percutaneous coronary intervention (PCI).
C. Obtain serial cardiac biomarkers.
D. Administer sublingual nitroglycerin every 5 minutes for up to 3 doses.
B. Prepare for immediate percutaneous coronary intervention (PCI).
Rationale: The presence of ST-segment elevations in leads V2-V4 suggests an anterior wall STEMI, likely due to a complete occlusion of the left anterior descending (LAD) artery. PCI is the gold standard for reperfusion therapy and should be performed within 90 minutes of first medical contact to minimize myocardial damage. Rapid restoration of blood flow prevents further ischemia and necrosis.
A nurse is providing care to a patient diagnosed with non-ST-segment elevation myocardial infarction (NSTEMI). Which of the following findings support this diagnosis? Select all that apply.
A. Elevated serum troponin levels
B. ST-segment elevation on ECG
C. T-wave inversion on ECG
D. Chest pain not relieved by nitroglycerin
E. Partial occlusion of a coronary artery
A. Elevated serum troponin levels
C. T-wave inversion on ECG
D. Chest pain not relieved by nitroglycerin
E. Partial occlusion of a coronary artery
Rationale: NSTEMI is characterized by elevated serum troponin levels (A), T-wave inversion on ECG (C), and chest pain that persists despite nitroglycerin (D). It results from a partial occlusion of a coronary artery (E), leading to ischemia but not full-thickness myocardial damage. Unlike STEMI, NSTEMI does not show ST-segment elevation on ECG.
A 70-year-old patient with a history of coronary artery disease presents with sudden onset of chest pain, dyspnea, and diaphoresis. The ECG reveals ST-segment elevation in leads II, III, and aVF. The provider orders an emergent fibrinolytic therapy. The nurse knows that which of the following is an absolute contraindication to this therapy?
A. History of ischemic stroke 10 years ago
B. Active peptic ulcer disease
C. Intracranial hemorrhage 3 months ago
D. Blood pressure of 170/90 mmHg
C. Intracranial hemorrhage 3 months ago
Rationale: A history of recent intracranial hemorrhage is an absolute contraindication to fibrinolytic therapy due to the high risk of fatal bleeding. Fibrinolytics, such as alteplase, dissolve thrombi but also increase the risk of life-threatening hemorrhage, especially in patients with recent cerebrovascular events.
A nurse is reviewing the ECG of a patient with acute coronary syndrome (ACS). Which ECG finding is most indicative of myocardial injury?
A. ST-segment depression
B. T-wave inversion
C. Q waves
D. ST-segment elevation
D. ST-segment elevation
Rationale: ST-segment elevation is the hallmark of acute myocardial injury and indicates ongoing damage to the myocardium due to prolonged ischemia. If untreated, this injury progresses to infarction, leading to myocardial necrosis.
A nurse is educating a new graduate nurse about the pathophysiology of acute coronary syndrome (ACS). Which of the following statements should be included? Select all that apply.
A. ACS occurs when a stable atherosclerotic plaque ruptures.
B. Platelet aggregation leads to thrombus formation.
C. Myocardial cells become hypoxic within 10 seconds of total coronary occlusion.
D. Myocardial infarction is always reversible with early treatment.
E. Ischemic heart cells remain viable for approximately 20 minutes.
A. ACS occurs when a stable atherosclerotic plaque ruptures.
B. Platelet aggregation leads to thrombus formation.
C. Myocardial cells become hypoxic within 10 seconds of total coronary occlusion.
E. Ischemic heart cells remain viable for approximately 20 minutes.
Rationale: ACS occurs when a stable plaque ruptures (A), leading to platelet aggregation and thrombus formation (B). Myocardial cells become hypoxic within 10 seconds of total occlusion (C) and remain viable for about 20 minutes before irreversible damage occurs (E). However, myocardial infarction may not always be reversible, even with early treatment.
A patient experiencing acute chest pain is found to have ST-segment elevations and is scheduled for emergency PCI. Which laboratory value should the nurse prioritize before the procedure?
A. Serum creatinine
B. Serum potassium
C. Hemoglobin level
D. Blood glucose
A. Serum creatinine
Rationale: Serum creatinine is critical before PCI because contrast dye used in the procedure can cause nephrotoxicity, particularly in patients with pre-existing kidney dysfunction. Assessing renal function helps determine the patient’s ability to tolerate contrast media.
A nurse is monitoring a patient recovering from a STEMI. Which finding suggests the development of a life-threatening complication?
A. New onset of a loud systolic murmur
B. Bradycardia of 58 bpm
C. Persistent nausea and vomiting
D. Oxygen saturation of 95% on room air
A. New onset of a loud systolic murmur
Rationale: A new systolic murmur after a STEMI suggests papillary muscle rupture or ventricular septal defect, both of which are life-threatening complications requiring immediate intervention.
A patient recovering from an NSTEMI is being discharged. The nurse is providing education on secondary prevention strategies. Which of the following should be included? Select all that apply.
A. Take aspirin daily.
B. Avoid all forms of exercise.
C. Follow a diet low in saturated fats.
D. Monitor for chest pain and report any recurrence.
E. Take nitroglycerin only if pain lasts more than 10 minutes.
A. Take aspirin daily.
C. Follow a diet low in saturated fats.
D. Monitor for chest pain and report any recurrence.
Rationale: Daily aspirin (A) helps prevent future thrombotic events. A heart-healthy diet (C) reduces cardiovascular risk. Monitoring for chest pain (D) is essential for early intervention if symptoms recur.
A patient with acute coronary syndrome is prescribed a beta-blocker. The nurse should monitor for which expected effect?
A. Increased myocardial oxygen demand
B. Decreased heart rate and blood pressure
C. Increased cardiac contractility
D. Increased risk of thrombosis
B. Decreased heart rate and blood pressure
Rationale: Beta-blockers reduce myocardial oxygen demand by lowering heart rate and blood pressure, decreasing workload on the heart, and reducing the risk of infarct expansion.
A patient with ACS reports ongoing chest pain. The nurse should implement which interventions? Select all that apply.
A. Administer supplemental oxygen if SpO2 <90%.
B. Provide sublingual nitroglycerin as prescribed.
C. Encourage deep breathing exercises.
D. Position the patient in a high-Fowler’s position.
E. Administer IV morphine if pain persists.
A. Administer supplemental oxygen if SpO2 <90%.
B. Provide sublingual nitroglycerin as prescribed.
D. Position the patient in a high-Fowler’s position.
E. Administer IV morphine if pain persists.
Rationale: Oxygen (A) improves myocardial oxygenation. Nitroglycerin (B) dilates coronary arteries. Positioning the patient upright (D) reduces cardiac workload. Morphine (E) helps relieve pain and anxiety, decreasing myocardial oxygen demand.
A 65-year-old male with a history of chronic stable angina presents to the emergency department reporting chest pain that began while he was watching television. He describes the pain as more intense than his usual episodes and states that it has lasted over 15 minutes. His ECG shows ST-segment depression in leads II, III, and aVF.
Which of the following interventions should the nurse anticipate first?
A. Obtain cardiac enzyme levels
B. Prepare the patient for immediate PCI
C. Administer sublingual nitroglycerin and aspirin
D. Schedule the patient for a treadmill stress test
C. Administer sublingual nitroglycerin and aspirin
Rationale: The patient’s chest pain at rest, prolonged duration, and ST-segment depression indicate unstable angina (UA), which requires immediate medical treatment to prevent myocardial infarction (MI). Sublingual nitroglycerin helps relieve ischemia by dilating coronary arteries and improving oxygen delivery, while aspirin prevents further platelet aggregation, reducing the risk of thrombus progression.
A nurse is educating a group of new nurses about unstable angina. Which statements should be included in the teaching? Select all that apply.
A. Unstable angina can occur at rest.
B. ST-segment elevation is a common finding in unstable angina.
C. Unstable angina pain typically lasts more than 10 minutes.
D. Patients with unstable angina may have ischemic ECG changes.
E. Unstable angina is always relieved with nitroglycerin.
A. Unstable angina can occur at rest.
C. Unstable angina pain typically lasts more than 10 minutes.
D. Patients with unstable angina may have ischemic ECG changes.
Rationale: Unstable angina can occur at rest (A), differentiating it from chronic stable angina. The pain typically lasts more than 10 minutes (C), unlike stable angina, which usually lasts 5 minutes or less and is relieved by rest or nitroglycerin. Ischemic ECG changes such as ST depression or T-wave inversion (D) may be present, but ST-segment elevation is not a hallmark of UA (B) and suggests a STEMI instead. UA is not always relieved by nitroglycerin (E), distinguishing it from stable angina.
A nurse is caring for a patient with suspected acute coronary syndrome (ACS). The patient reports chest pain that began 20 minutes ago while at rest and has not improved with sublingual nitroglycerin. The ECG shows ST depression in multiple leads. Which diagnostic test is most essential to differentiate between unstable angina (UA) and a myocardial infarction (MI)?
A. Serum troponin levels
B. Echocardiogram
C. Coronary angiography
D. Chest X-ray
A. Serum troponin levels
Rationale: Serum troponin levels are the key differentiating factor between unstable angina (UA) and myocardial infarction (MI). UA does not cause myocardial necrosis, so troponin levels remain normal, whereas an elevated troponin level indicates myocardial infarction due to irreversible cardiac muscle damage.
A 58-year-old female with no prior history of angina presents with chest discomfort that started three days ago and has progressively worsened. She now experiences pain even at rest. The ECG shows T-wave inversion in multiple leads. Which interventions should the nurse anticipate? Select all that apply.
A. Administering a heparin infusion
B. Providing supplemental oxygen if SpO₂ < 90%
C. Encouraging ambulation to assess for exertional chest pain
D. Administering a beta-blocker as prescribed
E. Preparing the patient for immediate fibrinolytic therapy
A. Administering a heparin infusion
B. Providing supplemental oxygen if SpO₂ < 90%
D. Administering a beta-blocker as prescribed
Rationale: Heparin (A) prevents clot formation, reducing the risk of progression to MI. Oxygen therapy (B) is recommended for SpO₂ <90% to improve oxygen delivery to ischemic myocardial tissue. Beta-blockers (D) reduce myocardial oxygen demand, decreasing the risk of infarction. Ambulation (C) is not recommended in a patient with active chest pain, and fibrinolytic therapy (E) is used for STEMI, not UA.
A nurse is caring for a patient admitted with unstable angina. The patient suddenly reports increasing chest pain and shortness of breath. The ECG shows worsening ST-segment depression. What is the nurse’s priority action?
A. Notify the healthcare provider immediately
B. Increase the patient’s IV fluid rate
C. Obtain a repeat 12-lead ECG and assess vital signs
D. Prepare the patient for emergency coronary artery bypass grafting (CABG)
C. Obtain a repeat 12-lead ECG and assess vital signs
Rationale: A worsening pattern of chest pain with increased ST-segment depression suggests worsening ischemia or progression to MI. The priority is to assess the patient’s current cardiac status with a repeat ECG and vital signs before notifying the provider. Immediate intervention is required if the patient is progressing to MI.
A nurse is reviewing risk factors for unstable angina (UA) progressing to myocardial infarction (MI). Which of the following increase the risk of MI in a patient with UA? Select all that apply.
A. History of diabetes mellitus
B. Presence of ST-segment elevation on ECG
C. Troponin elevation at baseline
D. Age over 65 years
E. Chronic kidney disease
A. History of diabetes mellitus
C. Troponin elevation at baseline
D. Age over 65 years
E. Chronic kidney disease
Rationale: Diabetes (A) increases the risk of atherosclerosis and silent ischemia. Elevated troponins at baseline (C) indicate ongoing myocardial injury, suggesting a high risk of MI. Older age (D) and chronic kidney disease (E) are known risk factors for cardiovascular events. ST-segment elevation (B) is indicative of STEMI, not UA.
A patient with unstable angina is receiving medical management. Which medication is most important in reducing myocardial oxygen demand and preventing infarction?
A. Nitroglycerin
B. Furosemide
C. Calcium channel blockers
D. Beta-blockers
D. Beta-blockers
Rationale: Beta-blockers are first-line therapy in UA to reduce myocardial oxygen demand by lowering heart rate, blood pressure, and contractility. This helps stabilize the myocardium and prevent infarction. Nitroglycerin (A) helps relieve acute chest pain but does not significantly reduce long-term oxygen demand. Furosemide (C) is not indicated unless heart failure is present. Calcium channel blockers (D) are used when beta-blockers are contraindicated.
A patient presents to the emergency department with chest pain. An ECG shows ST-segment elevation in leads II, III, and aVF. Which coronary artery is most likely occluded?
a. Left anterior descending (LAD) artery
b. Right coronary artery (RCA)
c. Left circumflex artery (LCX)
d. Posterior descending artery
b. Right coronary artery (RCA)
Rationale: ST-segment elevation in leads II, III, and aVF indicates an inferior wall MI, which is most commonly caused by occlusion of the RCA.
A patient is diagnosed with a STEMI and is in a hospital without a cardiac catheterization laboratory. Which intervention should the nurse anticipate?
a. Administration of thrombolytic therapy
b. Coronary artery bypass graft (CABG) surgery
c. Elective PCI scheduled within 24 hours
d. Continuous IV heparin infusion without reperfusion therapy
a. Administration of thrombolytic therapy
Rationale: In hospitals without PCI capabilities, thrombolytic (fibrinolytic) therapy is the preferred treatment to restore coronary artery blood flow within 90 minutes of symptom onset.
Which ECG finding suggests that a patient has had a previous myocardial infarction?
a. ST-segment elevation
b. T-wave inversion
c. Pathologic Q wave
d. Prolonged PR interval
c. Pathologic Q wave
Rationale: A pathologic Q wave (≥ one-third the height of the R wave in the same lead) is a hallmark of an old MI and indicates irreversible myocardial damage.
A patient with NSTEMI is admitted to the cardiac unit. Which of the following interventions is appropriate?
a. Immediate PCI
b. Administration of thrombolytics
c. Emergency CABG
d. Cardiac catheterization
d. Cardiac catheterization
Rationale: Unlike STEMI, NSTEMI is caused by a nonocclusive thrombus and does not require immediate PCI. However, cardiac catheterization is typically performed within 12-72 hours to assess and manage the blockage.
Which laboratory value is most indicative of myocardial infarction?
a. Elevated D-dimer
b. Decreased hemoglobin
c. Increased B-type natriuretic peptide (BNP)
d. Elevated troponin I
d. Elevated troponin I
Rationale: Troponin I is the most specific and sensitive biomarker for myocardial infarction and remains elevated for up to 10-14 days.
A patient with a STEMI asks why immediate PCI is necessary. What is the best response by the nurse?
a. “PCI helps reduce chest pain quickly.”
b. “PCI is the only way to confirm that you have had an MI.”
c. “PCI restores blood flow to your heart muscle to prevent further damage.”
d. “PCI eliminates the need for medications to manage your heart attack.”
c. “PCI restores blood flow to your heart muscle to prevent further damage.”
Rationale: PCI is the preferred treatment for STEMI because it rapidly reopens the blocked artery, restoring blood flow and minimizing myocardial damage.
A patient with a history of stable angina reports sudden chest pain at rest lasting over 10 minutes. What is the priority intervention?
a. Obtain a 12-lead ECG
b. Instruct the patient to take an antacid
c. Encourage deep breathing exercises
d. Apply a heating pad to the chest
a. Obtain a 12-lead ECG
Rationale: The patient’s symptoms suggest unstable angina or an acute coronary syndrome, requiring an immediate ECG to assess for ST-segment changes.
A patient with an anterior wall STEMI is at risk for which complication?
a. Right ventricular failure
b. Cardiogenic shock
c. Atrial fibrillation
d. Pulmonary embolism
b. Cardiogenic shock
Rationale: Anterior wall MIs, often caused by LAD occlusion, can lead to extensive LV damage and cardiogenic shock.
Which statement by the patient indicates an understanding of NSTEMI management?
a. “I will need emergency PCI within 90 minutes.”
b. “I should receive thrombolytic therapy right away.”
c. “I will have a heart catheterization within the next day or two.”
d. “NSTEMI is not as serious as a STEMI, so I don’t need treatment.”
c. “I will have a heart catheterization within the next day or two.”
Rationale: NSTEMI requires risk stratification and is typically managed with cardiac catheterization within 12-72 hours rather than emergency PCI.
A patient with an inferior wall STEMI is at risk for which of the following? Select all that apply.
a. Bradycardia
b. Cardiogenic shock
c. Right ventricular infarction
d. Atrial fibrillation
e. Papillary muscle dysfunction
a. Bradycardia
c. Right ventricular infarction
e. Papillary muscle dysfunction
Rationale: Inferior MIs often involve the RCA, which supplies the SA and AV nodes, leading to bradycardia. They are also associated with right ventricular infarctions and papillary muscle dysfunction, which can cause mitral regurgitation.
Which clinical finding is most concerning in a patient with a STEMI?
a. New-onset nausea and vomiting
b. ST-segment elevation in two contiguous leads
c. New S3 heart sound and hypotension
d. Troponin elevation above normal levels
c. New S3 heart sound and hypotension
Rationale: An S3 heart sound and hypotension suggest heart failure or cardiogenic shock, which are life-threatening complications of an MI.
A patient with a STEMI is receiving thrombolytic therapy. Which finding indicates the therapy is effective?
a. Persistent chest pain
b. Decreased troponin levels
c. Return of ST segment to baseline
d. New onset of premature ventricular contractions (PVCs)
c. Return of ST segment to baseline
Rationale: The resolution of ST-segment elevation suggests successful reperfusion of the occluded coronary artery.
A patient with a history of CAD presents with chest pain. What is the most appropriate question for the nurse to ask?
a. “Is the pain sharp or stabbing?”
b. “Have you taken any medication for the pain?”
c. “Do you feel pain in only one location?”
d. “Is the pain relieved by rest or nitroglycerin?”
d. “Is the pain relieved by rest or nitroglycerin?”
Rationale: This helps differentiate stable angina (relieved by rest or nitroglycerin) from unstable angina or MI, which is not relieved by these measures.
Which of the following should the nurse monitor closely in a patient receiving thrombolytic therapy for STEMI?
a. Hypertension
b. Bleeding
c. Bradycardia
d. Hyperkalemia
b. Bleeding
Rationale: Thrombolytic therapy carries a high risk of bleeding, including intracranial hemorrhage and GI bleeding.
A patient with an MI is found to have akinesis on echocardiography. What does this finding indicate?
a. Increased myocardial contractility
b. Absence of myocardial contractility
c. Atrial fibrillation
d. Increased collateral circulation
b. Absence of myocardial contractility
Rationale: Akinesis refers to absent myocardial contractility in the infarcted area, indicating irreversible myocardial damage.
A patient presents with chest pain and an ECG showing ST-segment depression. What is the priority intervention?
a. Administer aspirin and nitroglycerin
b. Prepare for immediate PCI
c. Administer thrombolytic therapy
d. Reassure the patient and monitor
a. Administer aspirin and nitroglycerin
Rationale: ST-segment depression suggests ischemia, which requires antiplatelet therapy (aspirin) and vasodilation (nitroglycerin) to improve blood flow.
Which ECG change suggests myocardial ischemia rather than infarction?
a. ST-segment elevation
b. T-wave inversion
c. Pathologic Q wave
d. Wide QRS complex
b. T-wave inversion
Rationale: T-wave inversion is a sign of myocardial ischemia, while ST-elevation and Q waves indicate infarction.
A patient with a history of NSTEMI is scheduled for a cardiac catheterization. Which statement by the patient indicates understanding of the procedure?
a. “I will receive clot-busting medication before the procedure.”
b. “This test will determine if I need a stent or bypass surgery.”
c. “I should avoid all fluids for 12 hours before the test.”
d. “This test will cure my coronary artery disease.”
b. “This test will determine if I need a stent or bypass surgery.”
Rationale: Cardiac catheterization helps assess coronary artery blockage and determine treatment options, such as PCI or CABG.
What is the main reason why younger individuals may experience more severe MIs than older individuals with the same blockage?
a. Younger individuals have higher cholesterol levels
b. Older individuals develop collateral circulation
c. Older individuals have lower blood pressure
d. Younger individuals have stronger immune responses
b. Older individuals develop collateral circulation
Rationale: Over time, individuals with CAD may develop collateral circulation, which can lessen the severity of an MI.
A patient with a STEMI is scheduled for PCI. Which action is a priority before the procedure?
a. Assess for aspirin allergy
b. Administer IV heparin
c. Ensure the patient is NPO for 12 hours
d. Hold all cardiac medications
a. Assess for aspirin allergy
Rationale: Aspirin is given before PCI to prevent platelet aggregation, so the nurse must assess for allergies.
Which complication should the nurse monitor for in a patient who had a large anterior wall MI?
a. Pulmonary embolism
b. Stroke
c. Left ventricular heart failure
d. Mitral valve prolapse
c. Left ventricular heart failure
Rationale: Anterior wall MIs affect the LV, increasing the risk of heart failure due to impaired myocardial contractility.
A patient presents with acute chest pain. Which symptom is most concerning for an acute MI?
a. Pain that worsens with deep inspiration
b. Chest pain relieved by nitroglycerin
c. Diaphoresis and a feeling of impending doom
d. Sharp pain localized to one small area
c. Diaphoresis and a feeling of impending doom
Rationale: Diaphoresis and a sense of impending doom are common symptoms of acute MI and indicate a need for immediate intervention.
A patient recovering from a STEMI asks how to reduce the risk of another heart attack. Which statement indicates the need for further teaching?
a. “I will take my aspirin every day.”
b. “I should avoid smoking and manage my cholesterol.”
c. “I don’t need to take my medications if I feel fine.”
d. “I will exercise and eat a heart-healthy diet.”
c. “I don’t need to take my medications if I feel fine.”
Rationale: Medications for CAD must be taken consistently to prevent another MI, even if the patient feels well.
A nurse is caring for a patient after PCI for a STEMI. Which finding requires immediate intervention?
a. Mild bruising at the insertion site
b. Pedal pulse weaker on the affected limb
c. Urine output of 200 mL in 4 hours
d. Patient reports back pain and hypotension
d. Patient reports back pain and hypotension
Rationale: Back pain and hypotension after PCI may indicate retroperitoneal bleeding, a medical emergency requiring immediate intervention.
A 58-year-old male presents to the emergency department with chest pain that he describes as a “crushing pressure” in the substernal area, radiating to his left arm. He reports the pain started 30 minutes ago while he was watching TV.
Which action should the nurse take first?
a. Administer sublingual nitroglycerin
b. Assess pain using a numerical scale
c. Draw blood for cardiac biomarkers
d. Obtain a 12-lead ECG
d. Obtain a 12-lead ECG
Rationale: The priority in suspected MI is obtaining an ECG to identify ST-segment changes that indicate ischemia or infarction. Early diagnosis guides immediate intervention.
A 72-year-old female with diabetes presents with fatigue and shortness of breath but denies chest pain. Her vital signs are BP 98/62 mmHg, HR 112 bpm, RR 24, and SpO₂ 93% on room air. The nurse should recognize that her symptoms may be indicative of:
a. A panic attack
b. A silent myocardial infarction
c. Gastroesophageal reflux disease (GERD)
d. Pulmonary embolism
b. A silent myocardial infarction
Rationale: Patients with diabetes often experience silent MIs due to cardiac neuropathy, leading to atypical symptoms such as fatigue and dyspnea instead of chest pain.
A 65-year-old male with a history of coronary artery disease presents with new-onset indigestion, nausea, and mild epigastric discomfort. He reports taking an antacid with no relief. What is the nurse’s priority action?
a. Reassure the patient that it is likely GERD
b. Obtain a 12-lead ECG
c. Encourage the patient to take another antacid
d. Assess for food intake history
b. Obtain a 12-lead ECG
Rationale: Epigastric discomfort unrelieved by antacids may indicate an MI. Patients, especially older adults, may misinterpret cardiac pain as indigestion, delaying treatment.
The nurse is assessing a 75-year-old patient who recently experienced an MI. The patient is confused and reports dizziness but denies any chest pain. Which assessment finding would concern the nurse the most?
a. Irregular apical pulse
b. New-onset shortness of breath
c. Decreased urine output
d. ST-segment elevation on ECG
d. ST-segment elevation on ECG
Rationale: ST-segment elevation indicates an acute MI, which is a medical emergency. Older adults may present with confusion, dizziness, or dyspnea instead of chest pain.
A 50-year-old woman comes to the clinic with complaints of extreme fatigue and shortness of breath for the past two days. She denies chest pain but states she “just doesn’t feel right.” Which action should the nurse take next?
a. Assess her for anxiety
b. Perform a detailed pain assessment
c. Obtain vital signs and a 12-lead ECG
d. Encourage her to rest and hydrate
c. Obtain vital signs and a 12-lead ECG
Rationale: Women may experience atypical MI symptoms, such as extreme fatigue and dyspnea, rather than classic chest pain. An ECG is necessary to rule out an MI.
The nurse is caring for a patient who had an MI in the early morning hours. The patient asks why MIs often occur in the morning. The nurse’s best response is:
a. “There are natural changes in hormone levels that increase clot formation risk in the morning.”
b. “People tend to be more active in the morning, which increases heart strain.”
c. “Blood pressure tends to be lower in the morning, reducing oxygen to the heart.”
d. “The body’s metabolism slows during sleep, making it harder for the heart to function in the morning.”
a. “There are natural changes in hormone levels that increase clot formation risk in the morning.”
Rationale: MIs often occur in the early morning due to increased catecholamine release, which raises blood pressure and heart rate, promoting plaque rupture and clot formation.
A 56-year-old male presents to the emergency department with chest pain. On assessment, the nurse notices the patient is diaphoretic, with cool, clammy, and ashen skin. His heart rate is 110 bpm, and his blood pressure is 140/90 mmHg. Which response is the nurse’s priority?
a. Administer a sedative to relieve anxiety
b. Start an intravenous (IV) line and administer fluids
c. Monitor the patient’s vital signs every 15 minutes
d. Recognize these symptoms as indicative of sympathetic nervous system stimulation
d. Recognize these symptoms as indicative of sympathetic nervous system stimulation
Rationale: Sympathetic nervous system stimulation due to MI leads to diaphoresis, tachycardia, and vasoconstriction, as evidenced by the patient’s symptoms. Recognizing these signs helps prioritize immediate care and monitoring for MI.
The nurse is caring for a patient in the acute phase of a myocardial infarction. The patient exhibits increased heart rate, elevated blood pressure, and cool, clammy skin. Which underlying mechanism is causing these symptoms?
a. Parasympathetic nervous system activation
b. Release of catecholamines by ischemic heart cells
c. Increased metabolic rate due to hypoxia
d. Decreased blood volume leading to vasoconstriction
b. Release of catecholamines by ischemic heart cells
Rationale: The ischemic heart cells release catecholamines (norepinephrine and epinephrine), which lead to tachycardia, elevated blood pressure, and vasoconstriction, as seen in the patient.
A nurse is monitoring a patient during the early phase of an MI. The patient exhibits signs of sympathetic nervous system stimulation, including tachycardia, diaphoresis, and cool, ashen skin. What should the nurse prioritize in managing this patient?
a. Administering pain relief and oxygen therapy
b. Monitoring the patient’s blood glucose level
c. Providing emotional support to alleviate anxiety
d. Increasing the patient’s fluid intake
a. Administering pain relief and oxygen therapy
Rationale: The priority in managing a patient with acute MI and sympathetic nervous system stimulation is addressing the ischemia (pain relief) and ensuring adequate oxygen supply to the heart muscle.
A 62-year-old male patient is admitted with a suspected myocardial infarction. Upon examination, the nurse notices crackles in the lungs and jugular venous distention (JVD). What is the most likely explanation for these findings?
a. Right ventricular dysfunction
b. Left ventricular dysfunction
c. Systemic hypotension
d. Pulmonary embolism
b. Left ventricular dysfunction
Rationale: Crackles in the lungs and JVD are common signs of left ventricular dysfunction. When the left ventricle fails to pump effectively, it leads to pulmonary congestion (crackles) and systemic congestion (JVD). These findings suggest that the left side of the heart is not effectively pumping blood.
A 58-year-old female patient with a history of heart disease develops edema, JVD, and hepatomegaly. What condition should the nurse most suspect?
a. Right ventricular dysfunction
b. Acute myocardial infarction
c. Left ventricular dysfunction
d. Pericarditis
a. Right ventricular dysfunction
Rationale: Right ventricular dysfunction is associated with systemic congestion, which leads to peripheral edema, JVD, and hepatic engorgement. The nurse should consider this as a possible cause when these signs are present in the patient.
A nurse assesses a 70-year-old male who has had a recent myocardial infarction. The patient’s blood pressure is low, and urine output has decreased. What is the likely cause of these findings?
a. Decreased cardiac output (CO)
b. Pulmonary edema
c. Infection from a catheter
d. Dehydration
a. Decreased cardiac output (CO)
Rationale: Low blood pressure and decreased urine output are signs of decreased cardiac output. In the setting of myocardial infarction, decreased CO can result from impaired heart function, leading to inadequate renal perfusion and reduced urine output.
A patient with a myocardial infarction develops a loud holosystolic murmur. What complication should the nurse suspect?
a. Aortic valve stenosis
b. Ventricular septal defect or papillary muscle rupture
c. Atrial septal defect
d. Mitral valve prolapse
b. Ventricular septal defect or papillary muscle rupture
Rationale: A loud holosystolic murmur following a myocardial infarction may indicate a ventricular septal defect or papillary muscle rupture. These complications can result in mitral regurgitation, where blood leaks back into the left atrium due to improper valve closure.
A 64-year-old patient with a recent myocardial infarction presents with an S3 and S4 heart sound. What is the most likely cause of these abnormal heart sounds?
a. Right ventricular dysfunction
b. Acute mitral valve prolapse
c. Left ventricular dysfunction
d. Pericardial effusion
c. Left ventricular dysfunction
Rationale: The presence of an S3 and S4 heart sound is suggestive of left ventricular dysfunction. These abnormal sounds indicate changes in ventricular filling pressures, commonly seen in heart failure or myocardial infarction, where the left ventricle is not functioning efficiently.
A patient with a myocardial infarction is experiencing hypotension and is developing oliguria. What should the nurse’s priority action be?
a. Administer a diuretic to relieve fluid overload
b. Increase intravenous fluids to improve renal perfusion
c. Monitor the patient’s blood pressure every hour
d. Administer pain medication to decrease the stress on the heart
b. Increase intravenous fluids to improve renal perfusion
Rationale: In the context of myocardial infarction and hypotension, oliguria is often a sign of decreased renal perfusion. The priority is to restore blood flow and improve renal perfusion by increasing IV fluids, which can help prevent acute renal failure.
A patient with an inferior wall myocardial infarction (MI) is experiencing nausea and vomiting. What is the most likely cause of these symptoms?
a. Reflex stimulation of the vomiting center by severe pain
b. Decreased renal perfusion
c. Vasovagal reflexes initiated by the infarcted heart muscle
d. Hypotension and reduced blood flow to the gastrointestinal tract
c. Vasovagal reflexes initiated by the infarcted heart muscle
Rationale: Nausea and vomiting in patients with inferior wall MIs are often caused by vasovagal reflexes, which are triggered by the infarcted heart muscle. This reflex affects the gastrointestinal system and can lead to nausea and vomiting.
A 55-year-old patient with a myocardial infarction complains of persistent nausea despite receiving pain medication. What is the most appropriate explanation for this symptom?
a. Side effect of narcotic pain medication
b. Increased intracranial pressure
c. Gastrointestinal bleeding from anticoagulant therapy
d. Vasovagal response due to heart muscle ischemia
d. Vasovagal response due to heart muscle ischemia
Rationale: Nausea and vomiting can be a result of vasovagal reflexes, especially in patients with inferior wall MIs. The reflex is triggered by ischemia in the heart muscle and can lead to gastrointestinal symptoms, including nausea.
A patient with an acute myocardial infarction (MI) is experiencing severe nausea and vomiting. The nurse recognizes that these symptoms may be a response to which of the following?
a. Increase in circulating catecholamines
b. Reflex stimulation of the vomiting center
c. Dehydration from excessive fluid loss
d. Low blood sugar levels due to stress
b. Reflex stimulation of the vomiting center
Rationale: Severe nausea and vomiting can be caused by reflex stimulation of the vomiting center in the brain, particularly during an acute MI, due to the intense pain and stress associated with the event.
A 62-year-old male with an inferior wall myocardial infarction reports feeling nauseated and vomiting. The nurse understands that this is most likely due to which mechanism?
a. Direct irritation of the stomach lining from ischemia
b. Reflex response mediated by the vomiting center
c. Use of narcotics for pain management
d. Acute gastrointestinal bleeding
b. Reflex response mediated by the vomiting center
Rationale: Nausea and vomiting in this patient are most likely due to a reflex response triggered by the infarcted heart muscle, particularly in inferior wall MIs. This reflex involves the vomiting center in the brain and leads to gastrointestinal symptoms.
A patient who recently suffered a myocardial infarction (MI) develops a fever of 101°F (38.3°C) within 24 hours of the event. What is the most likely cause of this fever?
a. Infection from an invasive procedure
b. Systemic inflammatory response due to myocardial cell death
c. Overuse of antipyretics
d. Severe dehydration from fluid loss
b. Systemic inflammatory response due to myocardial cell death
Rationale: A fever following a myocardial infarction is commonly caused by a systemic inflammatory response that results from the death of myocardial cells. This can increase the body’s temperature, often peaking within 24 to 48 hours post-MI.
A patient with a myocardial infarction (MI) is experiencing a fever of 100.6°F (38.1°C). The nurse explains that the fever is most likely due to which of the following?
a. Infection caused by the catheterization procedure
b. Inflammatory response to myocardial cell death
c. Malignant hyperthermia from anesthetic agents
d. Reaction to the use of thrombolytic therapy
b. Inflammatory response to myocardial cell death
Rationale: Fever in the setting of an MI is commonly due to the systemic inflammatory response caused by the death of myocardial cells. This type of fever typically appears within 24 to 48 hours and can last for several days.
A patient who is recovering from a myocardial infarction (MI) develops a fever within the first 48 hours after the event. The nurse should anticipate that the fever is most likely due to:
a. An infection related to prolonged immobility
b. A normal systemic inflammatory response to myocardial cell death
c. Allergic reaction to medication used during the procedure
d. Hypoxia due to decreased cardiac output
b. A normal systemic inflammatory response to myocardial cell death
Rationale: A fever occurring within the first 24 to 48 hours after an MI is typically a result of the body’s systemic inflammatory response to the death of myocardial cells. This type of fever is common and usually resolves within 4 to 5 days.
A patient is 4 days post-myocardial infarction (MI). The nurse notes that the necrotic muscle wall is thinning, and the patient’s ECG shows T-wave inversion. What is the likely explanation for these changes?
a. Neutrophils and macrophages are removing necrotic tissue
b. A new scar tissue is being formed in the infarcted area
c. The infarcted area is fully healed
d. Increased oxygen delivery to the myocardium
a. Neutrophils and macrophages are removing necrotic tissue
Rationale: Within 4 days after an MI, neutrophils and macrophages infiltrate the infarcted tissue to remove necrotic debris. This process weakens the muscle wall, which is why T-wave inversion is observed on the ECG.
A patient who is recovering from a myocardial infarction (MI) reports a high blood glucose level. The nurse knows this is likely due to which process following infarction?
a. Lipolysis and glycogenolysis mediated by catecholamines
b. Hyperglycemia as a result of corticosteroid use
c. Increased insulin sensitivity due to cell death
d. Decreased secretion of glucagon from the pancreas
a. Lipolysis and glycogenolysis mediated by catecholamines
Rationale: After an MI, catecholamines initiate lipolysis and glycogenolysis, which increase plasma glucose levels. The elevated glucose is used by the oxygen-deprived myocardium for anaerobic metabolism.
A patient 10 days post-myocardial infarction (MI) is assessed for signs of ventricular dysfunction. The nurse is aware that the infarcted area during this time is:
a. Healed and fully compliant with the surrounding myocardium
b. Replaced by strong scar tissue that prevents wall motion
c. Weak and vulnerable to increased stress
d. Completely restored to its normal function
c. Weak and vulnerable to increased stress
Rationale: At 10 to 14 days post-MI, the scar tissue is still weak, and the heart muscle is susceptible to increased stress. The patient’s activity level may increase, so careful monitoring is essential to avoid complications.
A patient is 6 weeks post-myocardial infarction (MI). The nurse would expect the infarcted area to be:
a. Fully healed with strong, functional muscle tissue
b. Partially healed with permanent conduction disturbances
c. Healed with scar tissue replacing necrotic muscle
d. Still undergoing necrosis with no tissue repair
c. Healed with scar tissue replacing necrotic muscle
Rationale: By 6 weeks post-MI, scar tissue replaces the necrotic muscle tissue, and the infarcted area is considered healed. However, the scarred area may be less compliant than the surrounding tissue, which can lead to abnormal wall motion and complications such as dysrhythmias.
A patient recovering from a myocardial infarction (MI) is assessed using echocardiography. The results show hypokinesis in the infarcted area. This finding suggests:
a. A fully healed infarction with normal myocardial function
b. Increased compliance of the myocardial tissue
c. Abnormal wall motion due to scar tissue formation
d. Complete restoration of myocardial contractility
c. Abnormal wall motion due to scar tissue formation
Rationale: Scar tissue that forms in the infarcted area may result in abnormal wall motion, such as hypokinesis or akinesis, leading to reduced myocardial contractility and possible heart failure.
Which of the following is the primary purpose of ACE inhibitors in a patient recovering from a myocardial infarction (MI)?
a. To increase the formation of scar tissue in the infarcted area
b. To prevent ventricular remodeling and heart failure
c. To reduce blood glucose levels post-MI
d. To reduce the risk of arrhythmias in the infarcted area
b. To prevent ventricular remodeling and heart failure
Rationale: ACE inhibitors are used after an MI to prevent ventricular remodeling, which can lead to heart failure. Ventricular remodeling is a process where normal myocardium hypertrophies and dilates to compensate for damaged tissue, but this can ultimately worsen heart function.
A nurse is assessing a patient 3 months after a myocardial infarction (MI). Which finding is most likely to occur in the area of the infarcted muscle?
a. Complete restoration of normal myocardial function
b. Scar tissue that is less compliant and may lead to abnormal wall motion
c. Full recovery of contractility in the infarcted region
d. No changes in the infarcted area
b. Scar tissue that is less compliant and may lead to abnormal wall motion
Rationale: By 3 months post-MI, the infarcted area will have scar tissue, which is less compliant than the surrounding myocardial tissue. This can result in abnormal wall motion, which may be identified through echocardiography or nuclear imaging.
A patient is being monitored for signs of ventricular remodeling after a myocardial infarction (MI). The nurse should be aware that remodeling is a compensatory mechanism that can lead to:
a. Improved cardiac function and decreased risk of heart failure
b. Complete recovery of myocardial function
c. Normal ventricular size and compliance
d. Increased risk of sudden cardiac death and heart failure
d. Increased risk of sudden cardiac death and heart failure
Rationale: Ventricular remodeling can lead to increased risk of heart failure and sudden cardiac death (SCD). As normal myocardium hypertrophies and dilates to compensate for infarcted tissue, it can cause abnormalities in cardiac function, such as dysrhythmias and decreased contractility.
After a myocardial infarction (MI), a patient’s heart undergoes ventricular remodeling. Which of the following treatments is most commonly used to limit this process?
a. Beta-blockers
b. ACE inhibitors
c. Anticoagulants
d. Statins
b. ACE inhibitors
Rationale: ACE inhibitors are used to limit ventricular remodeling after an MI by reducing the stress on the heart, which can help prevent the development of heart failure. They block the effects of angiotensin II, which is involved in the process of remodeling.
A nurse is explaining the healing process after a myocardial infarction (MI) to a patient. Which statement by the nurse is most accurate regarding the process of scar tissue formation?
a. “Scar tissue fully restores the infarcted area to normal function within a few weeks.”
b. “Scar tissue begins to form within the first 24 hours and is fully functional by 6 weeks.”
c. “Scar tissue takes 6 weeks to fully form, but it will not restore normal myocardial function.”
d. “Scar tissue is fully functional, and there will be no loss of heart function.”
c. “Scar tissue takes 6 weeks to fully form, but it will not restore normal myocardial function.”
Rationale: Scar tissue replaces necrotic tissue after an MI, but it is less compliant than the normal myocardium and does not restore the infarcted area to its previous functional capacity. This is why the area is more vulnerable to complications like dysrhythmias or heart failure.
A patient recovering from a myocardial infarction (MI) is at risk for developing heart failure due to ventricular remodeling. Which of the following is a key factor contributing to the development of heart failure in this patient?
a. Increased myocardial contractility
b. Decreased scar tissue formation
c. Compensatory hypertrophy and dilation of the myocardium
d. Normalization of heart function within 6 weeks
c. Compensatory hypertrophy and dilation of the myocardium
Rationale: Ventricular remodeling involves the hypertrophy and dilation of the normal myocardium to compensate for infarcted tissue. This process can eventually lead to heart failure, especially if multiple coronary arteries are affected.
A patient who is recovering from a myocardial infarction (MI) is found to have hypokinesis in the infarcted area during echocardiography. This suggests:
a. The heart muscle is fully healed and restored to normal function
b. The infarcted tissue is now compliant with the surrounding myocardium
c. The patient is experiencing a complication of heart failure
d. There is reduced myocardial contractility in the infarcted region
d. There is reduced myocardial contractility in the infarcted region
Rationale: Hypokinesis refers to reduced movement or contractility of the myocardium in the infarcted area. This is a common finding after MI due to the formation of scar tissue, which does not contract as effectively as healthy muscle tissue.
A patient is admitted to the hospital following a myocardial infarction (MI) and is experiencing premature ventricular contractions (PVCs). The nurse knows that these rhythms are most likely to occur:
a. After reperfusion therapy or percutaneous coronary intervention (PCI)
b. In the presence of complete heart block
c. During the prehospital period due to ischemia
d. With sustained ventricular tachycardia (VT)
a. After reperfusion therapy or percutaneous coronary intervention (PCI)
Rationale: PVCs are commonly seen after reperfusion therapy (thrombolytic therapy or PCI) and are generally not associated with an increased risk for sudden cardiac death (SCD), unless the patient is symptomatic.
A nurse is caring for a patient who is 2 hours post-myocardial infarction (MI). The patient is experiencing ventricular fibrillation (VF). What is the most likely cause of this dysrhythmia?
a. Reperfusion therapy
b. Ischemia affecting the heart cells’ sensitivity to nerve impulses
c. The destruction of the sinus node
d. Electrolyte imbalances due to medication
b. Ischemia affecting the heart cells’ sensitivity to nerve impulses
Rationale: Ventricular fibrillation (VF) most commonly occurs within the first 4 hours after the onset of MI due to ischemia affecting the heart’s electrical system. Ischemia leads to changes in the heart cells’ sensitivity to nerve impulses, increasing the risk for dysrhythmias.
A patient is diagnosed with a myocardial infarction (MI) and has been started on thrombolytic therapy. The nurse anticipates the patient may experience which of the following?
a. Bradycardia
b. Nonsustained ventricular tachycardia (VT)
c. Sinus tachycardia
d. Third-degree heart block
b. Nonsustained ventricular tachycardia (VT)
Rationale: After reperfusion (either with thrombolytic therapy or percutaneous coronary intervention), nonsustained VT, PVCs, and idioventricular rhythms are common. These are usually not treated unless the patient is symptomatic.
A patient is found to have complete heart block following a myocardial infarction (MI). The nurse should anticipate which of the following interventions?
a. Administering an antiarrhythmic drug
b. Starting intravenous lidocaine to treat the bradycardia
c. Monitoring for hyperkalemia and treating it immediately
d. Inserting an external pacemaker or a temporary transvenous pacemaker
d. Inserting an external pacemaker or a temporary transvenous pacemaker
Rationale: Complete heart block can occur when areas of the conduction system, such as the sinus or atrioventricular node, are destroyed during an MI. Symptomatic bradycardia caused by complete heart block may require an external pacemaker or a temporary transvenous pacemaker.
A patient is experiencing ventricular tachycardia (VT) within 2 hours of experiencing chest pain from a myocardial infarction (MI). What is the most appropriate immediate intervention for this patient?
a. Administering aspirin to prevent further ischemia
b. Starting defibrillation for sustained VT
c. Providing supplemental oxygen and monitoring the rhythm
d. Initiating pharmacologic treatment for bradycardia
b. Starting defibrillation for sustained VT
Rationale: VT is a potentially life-threatening dysrhythmia that can lead to sudden cardiac death. If the VT is sustained, immediate defibrillation is necessary to restore a normal rhythm.
A nurse is assessing a patient who is 3 hours post-myocardial infarction (MI) and notes the presence of premature ventricular contractions (PVCs). The nurse should:
a. Administer a bolus of potassium
b. Start intravenous magnesium sulfate
c. Reassure the patient, as PVCs are common in the early post-MI period
d. Prepare for emergency defibrillation
c. Reassure the patient, as PVCs are common in the early post-MI period
Rationale: PVCs are common in the early post-MI period, especially with reperfusion therapy or PCI, and are generally not dangerous unless the patient is symptomatic. Reassurance and monitoring are appropriate unless the patient becomes symptomatic.
A patient is 1 hour post-myocardial infarction (MI) and is showing signs of ventricular fibrillation (VF). Which of the following interventions should the nurse prioritize?
a. Administering thrombolytic therapy
b. Performing immediate defibrillation
c. Preparing for percutaneous coronary intervention (PCI)
d. Monitoring for signs of hypotension
b. Performing immediate defibrillation
Rationale: Ventricular fibrillation (VF) is a life-threatening dysrhythmia that requires immediate defibrillation to restore a normal rhythm and prevent death. Thrombolytic therapy and PCI are not interventions for VF.
A nurse is monitoring a patient for dysrhythmias following a myocardial infarction (MI). Which of the following is most likely to occur within the first 4 hours after the onset of chest pain?
a. Atrial fibrillation
b. Complete heart block
c. Supraventricular tachycardia
d. Ventricular tachycardia (VT)
d. Ventricular tachycardia (VT)
Rationale: Ventricular tachycardia (VT) is most likely to occur within the first 4 hours of an MI due to ischemia and electrolyte imbalances. This period is critical for monitoring and intervention to prevent sudden cardiac death.
A nurse is assessing a patient who is experiencing early left-sided heart failure. Which of the following findings is most indicative of left-sided heart failure?
a. Jugular venous distention (JVD)
b. Abdominal distention
c. Crackles on auscultation of the lungs
d. Hepatic congestion
c. Crackles on auscultation of the lungs
Rationale: Crackles on auscultation of the lungs are indicative of pulmonary congestion, a hallmark sign of left-sided heart failure. Left-sided heart failure causes fluid to back up into the lungs, leading to crackles. JVD, abdominal distention, and hepatic congestion are more indicative of right-sided heart failure.
A patient with heart failure reports increasing difficulty breathing when lying flat. This is an indication of which condition?
a. Pulmonary embolism
b. Paroxysmal nocturnal dyspnea (PND)
c. Orthostatic hypotension
d. Pneumothorax
b. Paroxysmal nocturnal dyspnea (PND)
Rationale: Paroxysmal nocturnal dyspnea (PND) is characterized by sudden difficulty breathing when lying flat, a common symptom of left-sided heart failure due to fluid accumulation in the lungs. Orthopnea, on the other hand, is difficulty breathing when lying down, but it is more gradual than PND. Pulmonary embolism and pneumothorax would have different presenting symptoms.
A nurse is assessing a patient with signs of right-sided heart failure. Which of the following findings is most likely in this patient?
a. Pulmonary congestion
b. Orthopnea
c. Lower extremity edema
d. S3 heart sound
c. Lower extremity edema
Rationale: Right-sided heart failure leads to fluid retention in the body, which commonly results in lower extremity edema, along with other signs such as abdominal distention and jugular venous distention (JVD). Pulmonary congestion and orthopnea are associated with left-sided heart failure. An S3 heart sound may also indicate left-sided heart failure.
A nurse notes that a patient with heart failure is becoming increasingly restless, agitated, and tachycardic. These symptoms are indicative of which stage of heart failure?
a. Severe right-sided heart failure
b. Early left-sided heart failure
c. Decompensated heart failure
d. End-stage heart failure
b. Early left-sided heart failure
Rationale: Restlessness, agitation, and tachycardia are early, subtle signs of left-sided heart failure as the body begins to compensate for reduced cardiac output. As heart failure progresses, symptoms become more severe, such as dyspnea, pulmonary congestion, and fluid retention.
A patient with left-sided heart failure is experiencing paroxysmal nocturnal dyspnea (PND). Which of the following is the most appropriate intervention to help relieve this symptom?
a. Encourage the patient to lie flat in bed
b. Administer a sedative to calm the patient
c. Elevate the head of the bed to facilitate breathing
d. Administer a diuretic to remove excess fluid
c. Elevate the head of the bed to facilitate breathing
Rationale: Elevating the head of the bed helps relieve the symptoms of paroxysmal nocturnal dyspnea (PND) by reducing the venous return to the heart and easing the patient’s breathing. Lying flat would worsen the symptoms, and administering a sedative or diuretic does not directly address the acute issue of fluid accumulation in the lungs.
A patient with heart failure develops crackles on auscultation and an S3 heart sound. What does this indicate?
a. Right-sided heart failure
b. Pulmonary embolism
c. Left-sided heart failure with pulmonary congestion
d. Exacerbation of stable heart failure
c. Left-sided heart failure with pulmonary congestion
Rationale: Crackles on auscultation and an S3 heart sound are classic signs of left-sided heart failure, which leads to pulmonary congestion as fluid backs up into the lungs. These findings indicate that the left side of the heart is not pumping effectively, resulting in fluid accumulation in the lungs.
A patient with a STEMI has developed cardiogenic shock. Which of the following is the most appropriate immediate intervention to support the patient’s condition?
a. Administer high doses of diuretics
b. Initiate mechanical circulatory support with an intraaortic balloon pump (IABP)
c. Administer broad-spectrum antibiotics
d. Perform percutaneous coronary intervention (PCI)
b. Initiate mechanical circulatory support with an intraaortic balloon pump (IABP)
Rationale: In cardiogenic shock, mechanical circulatory support, such as an intraaortic balloon pump (IABP), is used to improve myocardial perfusion and reduce the workload on the heart. Diuretics and antibiotics are not indicated in the acute management of cardiogenic shock, and while PCI may be performed, it is not the most immediate intervention in this acute setting.
A patient is diagnosed with cardiogenic shock following an acute STEMI. Which of the following goals of therapy should be the priority during the management of this patient?
a. Maximize oxygen delivery to tissues
b. Administer a sedative to reduce anxiety
c. Promote early ambulation
d. Limit fluid intake to reduce edema
a. Maximize oxygen delivery to tissues
Rationale: The primary goal in managing cardiogenic shock is to maximize oxygen delivery to tissues, as oxygen supply is inadequate due to severe left ventricular failure. This may involve oxygen therapy, optimizing cardiac output, and using vasoactive medications. Reducing anxiety, promoting ambulation, or limiting fluid intake are not immediate priorities in the acute management of cardiogenic shock.
A nurse is caring for a patient with cardiogenic shock who is receiving dobutamine. The patient begins to show signs of worsening arrhythmias. Which action should the nurse take first?
a. Decrease the rate of the dobutamine infusion
b. Administer an antidote for the dobutamine
c. Increase the dose of dobutamine
d. Discontinue the dobutamine infusion immediately
a. Decrease the rate of the dobutamine infusion
Rationale: Dobutamine is a vasoactive drug used to support contractility in cardiogenic shock, but it can cause arrhythmias as a side effect. The first action should be to decrease the rate of the infusion to mitigate the potential for worsening arrhythmias. Discontinuing the drug or increasing the dose may worsen the situation. Administering an antidote is not necessary for dobutamine.
A patient with cardiogenic shock requires mechanical circulatory support. Which of the following devices may be used to improve myocardial perfusion and reduce the workload on the heart?
a. Intraaortic balloon pump (IABP)
b. Endotracheal tube
c. Chest tube
d. Percutaneous coronary intervention (PCI)
a. Intraaortic balloon pump (IABP)
Rationale: An intraaortic balloon pump (IABP) is used in patients with cardiogenic shock to improve myocardial perfusion and reduce the workload of the heart by inflating and deflating in sync with the cardiac cycle. An endotracheal tube and chest tube are not used for circulatory support, and while PCI may be performed to treat STEMI, it does not serve the same purpose as mechanical circulatory devices like the IABP.
A nurse is caring for a patient in cardiogenic shock due to a left ventricular infarction. Which of the following complications should the nurse prioritize in monitoring the patient?
a. Acute kidney injury (AKI)
b. Pneumonia
c. Deep vein thrombosis (DVT)
d. Gastrointestinal bleeding
a. Acute kidney injury (AKI)
Rationale: In cardiogenic shock, the priority complication to monitor for is acute kidney injury (AKI) due to inadequate renal perfusion. The kidneys are highly sensitive to changes in blood flow and oxygen delivery, and AKI can further compromise the patient’s condition. Pneumonia, DVT, and gastrointestinal bleeding may occur but are not as immediately critical as AKI in the context of cardiogenic shock.
A patient with a recent myocardial infarction (MI) presents with a new systolic murmur at the heart apex. The nurse suspects papillary muscle dysfunction. Which of the following diagnostic tools will confirm the diagnosis?
a. Chest X-ray
b. Electrocardiogram (ECG)
c. Echocardiogram
d. Cardiac catheterization
c. Echocardiogram
Rationale: An echocardiogram is the most definitive diagnostic tool for confirming papillary muscle dysfunction, as it can visualize mitral valve movement and regurgitation. While an ECG can identify arrhythmias and chest X-ray may show signs of pulmonary edema, they do not confirm papillary muscle dysfunction. Cardiac catheterization is not typically used to diagnose this condition.
A patient with papillary muscle rupture following a myocardial infarction is showing signs of acute mitral regurgitation, including dyspnea, pulmonary edema, and decreased cardiac output. Which of the following treatments would the nurse anticipate for this condition?
a. Immediate anticoagulation therapy
b. Afterload reduction with nitroprusside (Nipride) and/or intra-aortic balloon pump (IABP) therapy
c. Administration of beta-blockers
d. Administration of corticosteroids
b. Afterload reduction with nitroprusside (Nipride) and/or intra-aortic balloon pump (IABP) therapy
Rationale: In papillary muscle rupture, afterload reduction with nitroprusside and/or IABP therapy helps manage acute mitral regurgitation by reducing the workload on the heart and improving cardiac output. Anticoagulation therapy, beta-blockers, and corticosteroids are not primary treatments for this life-threatening complication.
A patient with a myocardial infarction has developed papillary muscle rupture, leading to acute mitral valve regurgitation. Which of the following is a primary symptom of this condition?
a. Bradycardia
b. Diaphoresis
c. Pulmonary edema
d. Chest pain
c. Pulmonary edema
Rationale: Pulmonary edema is a primary symptom of acute mitral regurgitation caused by papillary muscle rupture. Blood backs up into the left atrium and lungs, leading to fluid accumulation. Bradycardia, diaphoresis, and chest pain are not specific signs of this condition.
A nurse is assessing a patient who is post-myocardial infarction and hears a new systolic murmur suggestive of mitral regurgitation at the heart apex. Which action should the nurse take next?
a. Administer diuretics to relieve pulmonary edema
b. Obtain an echocardiogram to confirm papillary muscle dysfunction
c. Place the patient on a low-sodium diet
d. Prepare the patient for immediate surgery
b. Obtain an echocardiogram to confirm papillary muscle dysfunction
Rationale: A new systolic murmur at the heart apex suggests mitral regurgitation, which could be caused by papillary muscle dysfunction. An echocardiogram is required to confirm the diagnosis before further treatment decisions, including surgery or medication, are made.
A patient with papillary muscle rupture following a myocardial infarction is deteriorating rapidly. The nurse expects the patient will require immediate surgery. Which of the following surgical interventions is most likely to be performed?
a. Coronary artery bypass graft (CABG)
b. Mitral valve repair or replacement
c. Valve replacement surgery for aortic stenosis
d. Left ventricular assist device (LVAD) implantation
b. Mitral valve repair or replacement
Rationale: In the case of papillary muscle rupture, the patient will require urgent mitral valve repair or replacement to address the acute mitral regurgitation and prevent further deterioration. CABG, LVAD implantation, and aortic valve replacement are not appropriate for this specific condition.
After surgery to repair a ruptured papillary muscle and mitigate acute mitral regurgitation, the patient is recovering in the ICU. Which of the following interventions will help manage the patient’s condition and prevent further complications?
a. Monitor for signs of infection at the surgical site
b. Restrict fluid intake to minimize fluid overload
c. Administer anti-arrhythmic medications to prevent atrial fibrillation
d. Perform frequent neurological assessments to monitor for stroke
a. Monitor for signs of infection at the surgical site
Rationale: After mitral valve surgery, infection at the surgical site is a major concern. While fluid management and monitoring for arrhythmias are important, infection control is paramount in the early postoperative period. Stroke risk may be higher in some patients, but monitoring for infection is the priority.
A patient with a myocardial infarction develops papillary muscle rupture, and the healthcare provider prescribes afterload reduction therapy with nitroprusside (Nipride). What is the primary goal of this treatment?
a. Increase heart rate to improve cardiac output
b. Decrease afterload to improve cardiac efficiency
c. Enhance platelet aggregation to prevent bleeding
d. Decrease blood pressure to reduce ischemia
b. Decrease afterload to improve cardiac efficiency
Rationale: Nitroprusside (Nipride) is a vasodilator used to decrease afterload in patients with acute mitral regurgitation due to papillary muscle rupture. By reducing afterload, it improves cardiac efficiency and helps the heart pump more effectively, thus improving cardiac output. It does not directly affect heart rate, platelet aggregation, or ischemia.
A patient with a recent anterior myocardial infarction (MI) develops a left ventricular aneurysm. Which of the following complications is the patient at the highest risk for?
a. Renal failure
b. Hemorrhagic stroke
c. Pulmonary embolism
d. Angina and dysrhythmias
d. Angina and dysrhythmias
Rationale: A left ventricular aneurysm can lead to complications such as heart failure, dysrhythmias, and angina due to the abnormal contraction and increased strain on the heart. It does not directly cause renal failure, hemorrhagic stroke, or pulmonary embolism.
A patient with a left ventricular aneurysm following an anterior myocardial infarction is prescribed anticoagulation therapy. What is the primary reason for this treatment?
a. To prevent ventricular rupture
b. To reduce the risk of embolic stroke due to thrombus formation
c. To lower the patient’s blood pressure
d. To control dysrhythmias
b. To reduce the risk of embolic stroke due to thrombus formation
Rationale: Anticoagulation therapy is recommended for patients with left ventricular aneurysms to prevent thrombus formation, which can increase the risk of embolic stroke. While anticoagulation may help in reducing the risk of some complications, it is not specifically used to prevent ventricular rupture, control blood pressure, or manage dysrhythmias.
A patient with a left ventricular aneurysm secondary to an anterior myocardial infarction is at risk for ventricular rupture. What is the most likely outcome if this complication occurs?
a. Sudden cardiac death
b. Severe hypotension and shock
c. Immediate relief from angina
d. Acute renal failure
a. Sudden cardiac death
Rationale: Ventricular rupture is a life-threatening complication of a left ventricular aneurysm and usually leads to sudden cardiac death due to the rapid loss of heart function. It does not lead to immediate relief from angina or renal failure. Severe hypotension and shock are possible outcomes but are typically part of the progression to sudden cardiac death.
A patient is diagnosed with a left ventricular aneurysm after an anterior myocardial infarction. Which of the following symptoms is most indicative of heart failure in this patient?
a. S3 heart sound and crackles on lung auscultation
b. Chest pain and dizziness
c. Hematuria and oliguria
d. Jugular venous distention and edema in the lower extremities
a. S3 heart sound and crackles on lung auscultation
Rationale: An S3 heart sound and crackles on lung auscultation are classic signs of heart failure. The S3 indicates volume overload in the left ventricle, while crackles suggest pulmonary congestion. While the other symptoms may also occur in various conditions, they are not as specifically indicative of heart failure in this case.
Which of the following is the most likely reason for the development of a left ventricular aneurysm following an anterior myocardial infarction?
a. Inflammation of the pericardium
b. A clot obstructing blood flow to the heart
c. Thinning and bulging of the infarcted heart wall during contraction
d. The formation of scar tissue in the heart wall
c. Thinning and bulging of the infarcted heart wall during contraction
Rationale: A left ventricular aneurysm occurs when the infarcted heart wall thins and bulges out during contraction. This weakening and expansion of the heart muscle can lead to the formation of the aneurysm. Inflammation of the pericardium, clot formation, and scar tissue formation are not the primary causes of left ventricular aneurysms.
A patient with an acute myocardial infarction (MI) develops a new loud systolic murmur. What is the most likely cause of this murmur?
a. Papillary muscle rupture
b. Ventricular septal wall rupture
c. Mitral valve prolapse
d. Pericardial friction rub
b. Ventricular septal wall rupture
Rationale: A new loud systolic murmur in a patient with acute MI is most likely a sign of a ventricular septal wall rupture. This condition involves a defect in the septum between the right and left ventricles, causing a murmur due to the abnormal blood flow. Papillary muscle rupture and mitral valve prolapse do not typically cause this type of murmur. A pericardial friction rub is more commonly associated with pericarditis.
A patient with a ventricular septal wall rupture after an acute myocardial infarction (MI) is experiencing signs of heart failure (HF) and cardiogenic shock. What is the primary intervention for this condition?
a. Administering thrombolytics to dissolve a clot
b. Immediate emergency repair, either surgically or percutaneously
c. Initiating long-term anticoagulation therapy
d. Monitoring blood pressure and heart rate for 24 hours
b. Immediate emergency repair, either surgically or percutaneously
Rationale: Ventricular septal wall rupture requires immediate repair, either surgically or percutaneously, to prevent rapid hemodynamic compromise and further deterioration. Thrombolytics are not indicated in this scenario, as the issue is a mechanical defect in the heart wall, not a clot. Long-term anticoagulation therapy is not the primary treatment for this complication. Monitoring vital signs without intervention would not address the immediate danger.
A patient with left ventricular free wall rupture following an acute myocardial infarction (MI) is in critical condition. Which of the following is the most likely outcome if the rupture is not treated immediately?
a. Reversible ventricular dysfunction
b. Complete recovery without long-term complications
c. Rapid hemodynamic compromise and death
d. Development of a stable arrhythmia
c. Rapid hemodynamic compromise and death
Rationale: Left ventricular free wall rupture is a life-threatening emergency that causes rapid hemodynamic compromise and death if not treated immediately. This rupture leads to massive blood loss into the pericardial space, resulting in a tamponade effect and shock. It is not reversible without surgical intervention, and death rates are high if the condition is not addressed promptly.
A 70-year-old female patient with an anterior myocardial infarction (MI) is at high risk for which of the following complications?
a. Right ventricular hypertrophy
b. Left ventricular free wall rupture
c. Pulmonary embolism
d. Aortic dissection
b. Left ventricular free wall rupture
Rationale: Left ventricular free wall rupture is more common in patients who are older, especially after their first MI and in females. This complication occurs due to the structural vulnerability of the heart wall following an infarction, particularly in those with anterior MIs. Right ventricular hypertrophy and pulmonary embolism are less directly related to this condition. Aortic dissection is a separate cardiovascular issue unrelated to MI.
Which of the following is a key clinical finding that suggests a left ventricular free wall rupture in a patient after an acute myocardial infarction (MI)?
a. New loud systolic murmur
b. Severe chest pain and diaphoresis
c. Rapid hemodynamic instability and shock
d. Decreased jugular venous pressure (JVP)
c. Rapid hemodynamic instability and shock
Rationale: Left ventricular free wall rupture causes rapid hemodynamic compromise, leading to shock and collapse. This is a life-threatening condition that must be addressed immediately. A new loud systolic murmur is more indicative of a ventricular septal wall rupture. Severe chest pain and diaphoresis may occur with any MI, but they are not specific to a free wall rupture. Decreased JVP is not a typical finding in this scenario.
A patient with an anterior myocardial infarction (MI) is at increased risk for which of the following complications?
a. Right ventricular hypertrophy
b. Papillary muscle dysfunction
c. Left ventricular aneurysm
d. Aortic stenosis
c. Left ventricular aneurysm
Rationale: Patients with anterior MIs are at higher risk for developing left ventricular aneurysms due to the thinning and weakening of the infarcted area. This complication can result in heart failure, dysrhythmias, and angina. Right ventricular hypertrophy and papillary muscle dysfunction are possible but less common. Aortic stenosis is unrelated to MI complications.
A patient with a ventricular septal wall rupture after a recent MI is showing signs of hemodynamic instability. Which of the following is the most critical next step in management?
a. Initiating beta-blocker therapy
b. Administering high-dose diuretics
c. Immediate surgical or percutaneous repair of the defect
d. Observation in an intensive care unit for 24 hours
c. Immediate surgical or percutaneous repair of the defect
Rationale: Ventricular septal wall rupture requires immediate intervention with either surgical or percutaneous repair to prevent rapid hemodynamic compromise and further deterioration. Beta-blockers and diuretics are not appropriate in this emergency situation, as they do not address the underlying mechanical defect. Observation without intervention would lead to worsening shock and potential death.
A patient with acute pericarditis following an acute myocardial infarction (MI) reports chest pain that increases with inspiration, coughing, and movement. What is the most appropriate position to alleviate the pain?
a. Lying flat in bed
b. Sitting forward
c. Lying on the left side
d. Lying on the back with knees bent
b. Sitting forward
Rationale: Sitting in a forward position often relieves the chest pain associated with acute pericarditis. This position helps reduce pressure on the inflamed pericardium, thereby easing the pain. Lying flat or on the left side typically exacerbates the discomfort.
A patient with acute pericarditis following an acute MI develops hypotension and a narrow pulse pressure. What is the most likely cause of these findings?
a. Cardiac tamponade or significant pericardial effusion
b. Left ventricular failure or significant pericardial effusion
c. Pulmonary embolism or significant pericardial effusion
d. Anemia or significant pericardial effusion
a. Cardiac tamponade or significant pericardial effusion
Rationale: Hypotension and a narrow pulse pressure are common signs of cardiac tamponade, which occurs when pericardial effusion compresses the heart, restricting its ability to pump effectively. This can be a complication of acute pericarditis. Left ventricular failure and pulmonary embolism would present with different clinical findings.
A patient with acute pericarditis following an MI undergoes a 12-lead ECG. Which of the following ECG changes is most consistent with pericarditis?
a. ST segment elevation localized to the leads facing the infarcted wall
b. Q waves in the precordial leads
c. Diffuse ST segment elevation in many unrelated leads
d. Inverted T waves in leads V1-V3
c. Diffuse ST segment elevation in many unrelated leads
Rationale: Acute pericarditis is characterized by diffuse ST segment elevation across multiple leads that are unrelated to the infarcted wall. This differs from ST elevation in STEMI, where the elevation is localized to the leads facing the infarcted area. Q waves and inverted T waves suggest previous infarction, not pericarditis.
A patient with acute pericarditis is being treated with high doses of aspirin for pain relief. Why is it important to avoid the use of nonsteroidal anti-inflammatory drugs (NSAIDs) and corticosteroids within the first 4 weeks after a myocardial infarction (MI)?
a. NSAIDs and corticosteroids interfere with myocardial scar formation
b. They increase the risk of bleeding complications
c. They can cause excessive blood pressure elevation
d. They interfere with anticoagulation therapy
a. NSAIDs and corticosteroids interfere with myocardial scar formation
Rationale: NSAIDs and corticosteroids should be avoided in the first 4 weeks after an MI because they can interfere with the healing process of the myocardial scar, which is essential for the heart’s recovery after an infarction. They do not significantly affect bleeding, blood pressure, or anticoagulation therapy in this context.
A patient with acute pericarditis following a myocardial infarction is experiencing chest pain, fever, and a friction rub on auscultation. What is the most appropriate initial treatment for this patient?
a. Antibiotics
b. Corticosteroids
c. Beta-blockers
d. High-dose aspirin
d. High-dose aspirin
Rationale: High-dose aspirin is the treatment of choice for acute pericarditis to relieve pain and reduce inflammation. Corticosteroids and NSAIDs are avoided in the first 4 weeks post-MI due to their potential interference with scar formation. Antibiotics are not indicated unless there is evidence of infection, and beta-blockers do not directly address the inflammation.
A 60-year-old patient with a history of STEMI presents with a friction rub on auscultation. The patient is also experiencing mild to severe chest pain that increases with inspiration. What additional assessment should the nurse perform to help diagnose pericarditis?
a. Measure blood pressure for signs of hypertension
b. Check for jugular venous distention
c. Assess oxygen saturation levels
d. Perform a 12-lead ECG
d. Perform a 12-lead ECG
Rationale: A 12-lead ECG is essential for diagnosing pericarditis. It shows diffuse ST segment elevation across multiple leads, a key sign of pericardial inflammation. Blood pressure, jugular venous distention, and oxygen saturation are important in other contexts but are not diagnostic for pericarditis.
A patient diagnosed with acute pericarditis following an MI has mild hypotension and tachycardia. What would the nurse expect to hear upon auscultation of the heart?
a. An S3 heart sound
b. A friction rub
c. A loud systolic murmur
d. A fourth heart sound
b. A friction rub
Rationale: A friction rub is characteristic of acute pericarditis and is caused by the inflamed pericardium rubbing against the heart. It can be heard best with the diaphragm of the stethoscope at the left sternal border. An S3 or S4 heart sound would indicate heart failure or volume overload, not pericarditis. A systolic murmur is more commonly associated with valve defects.
A patient with acute pericarditis following an acute myocardial infarction (MI) presents with fever and chest pain that worsens with deep breathing. What should the nurse assess for next in this patient?
a. Tachycardia and widened pulse pressure
b. Jugular venous distention and peripheral edema
c. Hypotension and narrow pulse pressure
d. Hyperactive bowel sounds
c. Hypotension and narrow pulse pressure
Rationale: Hypotension and narrow pulse pressure may indicate the presence of pericardial effusion or cardiac tamponade, complications that can arise from pericarditis. Monitoring for these signs is critical in managing the patient’s condition. Jugular venous distention and peripheral edema are more commonly associated with heart failure, and hyperactive bowel sounds are unrelated to pericarditis.
A patient diagnosed with Dressler syndrome 3 weeks after a myocardial infarction (MI) presents with chest pain, fever, malaise, and a pericardial friction rub. What is the most likely cause of this syndrome?
a. Infection
b. Autoimmune reaction to necrotic heart muscle
c. Pulmonary embolism
d. Acute left ventricular failure
b. Autoimmune reaction to necrotic heart muscle
Rationale: Dressler syndrome is believed to be an autoimmune response to the necrotic heart tissue following an MI. It presents with symptoms of pericarditis, including chest pain, fever, and a friction rub. It is not caused by infection, pulmonary embolism, or heart failure.
A patient with Dressler syndrome presents with an elevated white blood cell count and sedimentation rate. These lab findings are most indicative of which of the following?
a. Infection
b. Inflammation
c. Renal failure
d. Hemorrhage
b. Inflammation
Rationale: The elevated white blood cell count and sedimentation rate are indicative of inflammation, which is a hallmark of Dressler syndrome, an autoimmune condition that results in pericarditis and fever. These lab findings are not typically associated with infection, renal failure, or hemorrhage in this context.
A patient diagnosed with Dressler syndrome following a myocardial infarction is experiencing fever and malaise. What is the recommended treatment for this condition?
a. High-dose aspirin
b. Anticoagulation therapy
c. Corticosteroids
d. Antibiotic therapy
a. High-dose aspirin
Rationale: High-dose aspirin is the treatment of choice for Dressler syndrome to manage inflammation and pain. Corticosteroids are avoided due to potential interference with myocardial healing, and antibiotics are not appropriate unless there is a secondary infection. Anticoagulation therapy is not indicated for Dressler syndrome.
A patient with Dressler syndrome develops a pericardial effusion that is visible on echocardiogram. What is the most appropriate next step in managing this patient?
a. Administering IV fluids
b. Performing a pericardiocentesis
c. Starting anticoagulation therapy
d. Initiating corticosteroid therapy
b. Performing a pericardiocentesis
Rationale: If a pericardial effusion is present and causing significant hemodynamic compromise, pericardiocentesis may be necessary to remove the fluid. The primary treatment for Dressler syndrome is high-dose aspirin, but pericardiocentesis is indicated if the effusion is large or symptomatic. Anticoagulation and corticosteroids are not appropriate for this situation.
A patient with Dressler syndrome develops joint pain in addition to the typical symptoms of chest pain, fever, and malaise. Which of the following clinical findings is consistent with Dressler syndrome?
a. Jaundice
b. Shortness of breath
c. Arthralgia
d. Syncope
c. Arthralgia
Rationale: Arthralgia (joint pain) is a common symptom of Dressler syndrome and is often seen alongside chest pain, fever, and malaise. This syndrome can involve multiple organ systems due to the inflammatory response, but jaundice, shortness of breath, and syncope are not typical findings of Dressler syndrome.
A patient presents with chest pain and risk factors for cardiovascular disease. Which diagnostic study is considered essential for determining whether the patient is experiencing unstable angina (UA) or a myocardial infarction (MI)?
a. Chest X-ray
b. 12-lead ECG
c. Pulmonary function test
d. Abdominal ultrasound
b. 12-lead ECG
Rationale: The 12-lead ECG is an essential diagnostic tool in determining whether a patient is experiencing unstable angina (UA) or a myocardial infarction (MI), including STEMI or NSTEMI. It provides critical information about the electrical activity of the heart, which helps in diagnosing these conditions. A chest X-ray, pulmonary function test, and abdominal ultrasound are not primary diagnostic studies for UA or MI.
Along with a 12-lead ECG, which of the following is a primary diagnostic study used to assess for myocardial infarction (MI)?
a. Serum cardiac biomarkers
b. Complete blood count (CBC)
c. B-type natriuretic peptide (BNP)
d. Chest X-ray
a. Serum cardiac biomarkers
Rationale: Serum cardiac biomarkers, such as troponins, are critical for diagnosing myocardial infarction (MI) when used alongside a 12-lead ECG. These biomarkers indicate heart muscle damage. While a CBC and BNP can be useful in other settings, they are not primary studies for diagnosing MI. Chest X-rays are typically used to evaluate lung conditions but do not directly assess myocardial infarction.
Which ECG finding is indicative of a STEMI?
a. ST segment depression in the leads facing the infarcted wall
b. T wave inversion in leads facing the ischemic area
c. ST segment elevation in the leads facing the infarcted wall
d. Pathologic Q waves in leads facing the infarcted area
c. ST segment elevation in the leads facing the infarcted wall
Rationale: ST segment elevation in the leads facing the infarcted wall is characteristic of STEMI. It indicates a complete coronary occlusion. This is the key finding for STEMI, whereas other findings like ST depression and T wave inversion are more commonly seen in NSTEMI or unstable angina.
How do ECG changes in a patient with NSTEMI differ from those with STEMI?
a. NSTEMI patients will have ST elevation and pathologic Q waves
b. NSTEMI patients will have normal ECG findings
c. NSTEMI patients will have no change in their T waves
d. NSTEMI patients will have ST depression and/or T wave inversion
d. NSTEMI patients will have ST depression and/or T wave inversion
Rationale: NSTEMI patients typically show ST depression and/or T wave inversion in the leads facing the area of ischemia or infarction. Unlike STEMI, NSTEMI does not involve complete coronary occlusion, and thus does not show ST elevation or pathologic Q waves.
What is the primary difference between the ECG findings in NSTEMI and unstable angina (UA)?
a. NSTEMI shows ST elevation, while UA shows T wave inversion
b. Both NSTEMI and UA show ST depression, but NSTEMI has elevated cardiac biomarkers
c. UA shows ST elevation, while NSTEMI shows T wave inversion
d. NSTEMI and UA both show pathologic Q waves
b. Both NSTEMI and UA show ST depression, but NSTEMI has elevated cardiac biomarkers
Rationale: The key distinction between NSTEMI and unstable angina is that NSTEMI involves elevated cardiac biomarkers due to tissue damage, while UA does not. Both conditions can present with ST depression, but the biomarkers differentiate the two.
In the setting of STEMI, which of the following is a hallmark of a complete coronary occlusion?
a. ST segment depression in the leads facing the infarcted wall
b. T wave inversion in the leads facing the infarcted wall
c. ST segment elevation in the leads facing the infarcted wall
d. Pathologic Q waves in the leads facing the infarcted wall
c. ST segment elevation in the leads facing the infarcted wall
Rationale: ST segment elevation in the leads facing the infarcted wall is indicative of a complete coronary occlusion, which is typical in STEMI. This finding is key for identifying STEMI and differentiating it from NSTEMI or UA.
What ECG change is most commonly seen when reperfusion is delayed or not performed in STEMI?
a. ST elevation
b. T wave inversion
c. Pathologic Q waves
d. ST depression
c. Pathologic Q waves
Rationale: Pathologic Q waves develop in the leads facing the infarcted wall when reperfusion is delayed or not performed in STEMI. These waves typically remain on the ECG forever, marking the area of infarction.
When is a 12-lead ECG most useful in diagnosing STEMI?
a. When T wave inversion is present in the leads facing the infarcted wall
b. When ST elevation is seen in the leads facing the infarcted wall
c. When pathologic Q waves are observed in the leads facing the infarcted wall
d. When cardiac biomarkers are high in the leads facing the infarcted wall
b. When ST elevation is seen in the leads facing the infarcted wall
Rationale: ST elevation in the leads facing the infarcted wall is the most definitive ECG finding for diagnosing STEMI. It indicates a complete coronary occlusion, which is a hallmark of STEMI.
What is the clinical significance of pathologic Q waves on an ECG?
a. They indicate ischemia in the heart muscle
b. They suggest acute myocardial infarction and remain permanently after an infarct
c. They are a sign of unstable angina
d. They indicate that reperfusion therapy was successful
b. They suggest acute myocardial infarction and remain permanently after an infarct
Rationale: Pathologic Q waves suggest an acute myocardial infarction (MI) and will remain permanently on the ECG. They indicate a large, irreversible infarction in the heart muscle.
What is the primary method used to distinguish between STEMI and NSTEMI/UA on the ECG?
a. Presence of pathologic Q waves
b. ST segment changes in the leads facing the infarcted area
c. T wave inversion in leads facing the ischemic area
d. Cardiac biomarkers
d. Cardiac biomarkers
Rationale: The presence of elevated cardiac biomarkers in NSTEMI is the key differentiator from UA, which has normal biomarkers. While both conditions can have similar ECG changes, cardiac biomarkers are essential in distinguishing between NSTEMI and UA.
How long after an acute MI can T wave inversion persist in a STEMI patient?
a. A few minutes
b. A few days
c. A few weeks
d. Several months
d. Several months
Rationale: T wave inversion can persist for several months after a STEMI, especially if reperfusion has been delayed or not performed. These changes indicate ongoing myocardial injury and recovery.
Which of the following ECG findings would be expected in a patient with unstable angina (UA)?
a. ST segment elevation
b. Pathologic Q waves
c. T wave inversion and/or ST depression
d. No changes in the ECG
c. T wave inversion and/or ST depression
Rationale: Unstable angina often presents with transient ECG changes, including T wave inversion and/or ST depression. However, the patient does not have elevated biomarkers as seen in NSTEMI, which differentiates UA from NSTEMI.
What is the significance of serial 12-lead ECGs in managing ACS?
a. To monitor the patient’s heart rate, injury, infarction, and resolution of the infarction
b. To track the evolution of ischemia, injury, infarction, and resolution of the infarction
c. To confirm a diagnosis of NSTEMI, injury, infarction, and resolution of the infarction
d. To assess the effectiveness of fibrinolytic therapy, injury, infarction, and resolution of the infarction
b. To track the evolution of ischemia, injury, infarction, and resolution of the infarction
Rationale: Serial 12-lead ECGs are used to monitor the progression of ACS, including the evolution of ischemia, injury, infarction, and resolution of the infarction. This helps guide treatment decisions and assess the effectiveness of interventions.
In the context of MI, which of the following ECG findings would suggest reperfusion therapy has been successful?
a. Persistent ST elevation
b. Reversal of T wave inversion
c. Resolution of ST elevation
d. New pathologic Q waves
c. Resolution of ST elevation
Rationale: The resolution of ST elevation is an indicator that reperfusion therapy has been successful. ST elevation is seen in STEMI due to complete coronary occlusion, and its resolution signals that the blockage has been cleared.
How can an ECG help in determining the coronary artery involved in ACS?
a. By identifying the location of ST elevation and pathologic Q waves
b. By detecting the size of the infarction
c. By evaluating the patient’s cardiac biomarkers
d. By determining the patient’s heart rate
a. By identifying the location of ST elevation and pathologic Q waves
Rationale: The location of ST elevation and pathologic Q waves on the ECG provides information about which coronary artery is involved in the ACS. These changes occur in specific leads that correspond to the distribution of blood flow from the affected artery.
What is the primary clinical utility of serum cardiac biomarkers in the diagnosis of MI?
a. To detect heart rate irregularities
b. To identify the coronary artery involved
c. To distinguish between UA and NSTEMI
d. To monitor heart failure progression
c. To distinguish between UA and NSTEMI
Rationale: Serum cardiac biomarkers, particularly troponins, are used to differentiate between unstable angina (UA) and NSTEMI. UA typically has negative biomarkers, while NSTEMI has elevated biomarkers due to myocardial injury.
What is the primary advantage of high-sensitivity cardiac troponin (hs-cTn) testing compared to conventional troponin assays?
a. It is more specific to heart muscle damage
b. It allows for faster detection of myocardial injury
c. It has a longer detection window
d. It is less expensive
b. It allows for faster detection of myocardial injury
Rationale: High-sensitivity cardiac troponin (hs-cTn) testing enables more rapid detection of myocardial injury compared to conventional troponin assays, providing quicker diagnosis and intervention.
When should a second high-sensitivity cardiac troponin (hs-cTn) be drawn in the setting of suspected STEMI?
a. 12 hours after the first test
b. 2 to 3 hours after the first test
c. 24 hours after the first test
d. Within 1 hour after the first test
b. 2 to 3 hours after the first test
Rationale: A second hs-cTn is typically drawn 2 to 3 hours after the first one if STEMI is ruled out, helping to confirm the diagnosis of myocardial injury or identify other potential causes.
Which of the following cardiac biomarkers is the most specific for diagnosing myocardial injury?
a. CK-MB
b. B-type natriuretic peptide (BNP)
c. Myoglobin
d. Cardiac-specific troponins (cTnI and cTnT)
d. Cardiac-specific troponins (cTnI and cTnT)
Rationale: Cardiac-specific troponins, including cTnI and cTnT, are the most specific biomarkers for diagnosing myocardial injury, as they are released exclusively from heart muscle cells.
How long do cardiac-specific troponins typically remain elevated after an MI?
a. 1 to 3 days
b. 3 to 7 days
c. 7 to 14 days
d. 1 to 2 months
c. 7 to 14 days
Rationale: Cardiac-specific troponins, including cTnI and cTnT, typically remain elevated for 7 to 14 days after an MI, making them useful for diagnosing MI even after the initial injury.
What is the main limitation of CK-MB as a biomarker for myocardial injury?
a. It has a long detection window
b. It is not specific to myocardial injury
c. It peaks later than troponins
d. It is expensive
b. It is not specific to myocardial injury
Rationale: CK-MB is less specific than cardiac-specific troponins because it is also found in other tissues, such as skeletal muscle. As a result, CK-MB is not as reliable for diagnosing myocardial injury compared to troponins.
How long after an MI do CK levels typically return to normal?
a. 12 to 24 hours
b. 24 to 36 hours
c. 48 to 72 hours
d. 1 week
b. 24 to 36 hours
Rationale: CK levels typically rise about 6 hours after an MI, peak at around 18 hours, and return to normal within 24 to 36 hours, making it a useful early marker for myocardial injury but less reliable for long-term diagnosis.
What is the role of serial cardiac-specific troponin measurements in the diagnosis of MI?
a. To monitor the resolution of infarction
b. To confirm the presence of a STEMI
c. To track the progression of heart failure
d. To detect the onset of arrhythmias
a. To monitor the resolution of infarction
Rationale: Serial cardiac-specific troponin measurements are used to monitor the resolution of infarction. These tests help track the peak levels and return to baseline, which indicates the progression or resolution of myocardial injury.
Which of the following is true about CK-MB as a myocardial injury biomarker?
a. CK-MB is the most specific cardiac biomarker
b. CK-MB levels remain elevated for several weeks after an MI
c. CK-MB is only elevated in patients with STEMI
d. CK-MB levels begin to rise 6 hours after an MI
d. CK-MB levels begin to rise 6 hours after an MI
Rationale: CK-MB levels begin to rise around 6 hours after an MI, peak at 18 hours, and return to baseline within 24 to 36 hours. While it is a useful early marker, it is less specific than cardiac-specific troponins.
What is the recommended time frame for performing cardiac catheterization in a patient with STEMI upon arrival at the emergency department?
a. Within 30 minutes
b. Within 60 minutes
c. Within 90 minutes
d. Within 120 minutes
c. Within 90 minutes
Rationale: The standard guideline for STEMI management is to perform cardiac catheterization within 90 minutes of arrival at the emergency department to restore blood flow and limit infarct size.
If a hospital does not have cardiac cath lab capability, what is the recommended alternative treatment for a STEMI patient?
a. CABG surgery within 12 hours
b. Thrombolytic therapy within 30 minutes
c. Medical management with beta-blockers
d. Cardiac catheterization within 24 hours
b. Thrombolytic therapy within 30 minutes
Rationale: If a hospital lacks PCI capability, guidelines recommend administering thrombolytic therapy within 30 minutes of arrival to dissolve the clot and restore perfusion.
A patient with NSTEMI is admitted to the hospital. When is cardiac catheterization typically recommended in this case?
a. Within 30 minutes of hospital arrival
b. Within 90 minutes of hospital arrival
c. Within 12 to 72 hours of presentation
d. Only if medical therapy fails
c. Within 12 to 72 hours of presentation
Rationale: Patients with NSTEMI or UA do not require emergent catheterization but are recommended to undergo the procedure within 12 to 72 hours to assess the severity of coronary artery disease and determine appropriate intervention.
What is the primary goal of performing cardiac catheterization in a patient with STEMI?
a. To evaluate the extent of heart failure
b. To diagnose coronary artery disease
c. To determine the need for long-term anticoagulation
d. To open the totally occluded artery and limit infarct size
d. To open the totally occluded artery and limit infarct size
Rationale: In STEMI patients, cardiac catheterization aims to rapidly open the occluded coronary artery and minimize myocardial infarction damage.
After cardiac catheterization for NSTEMI, what factors determine whether the patient will undergo PCI, CABG, or medical management?
a. The findings from the catheterization and patient factors
b. The patient’s age and gender
c. The hospital’s preference for treatment methods
d. Whether the patient has a history of smoking
a. The findings from the catheterization and patient factors
Rationale: The choice between PCI, CABG, or medical management depends on the severity of coronary artery disease observed during catheterization and individual patient factors such as comorbidities and surgical risk.
Why do guidelines recommend that patients with UA or NSTEMI do not require emergent catheterization like STEMI patients?
a. Because they are usually hemodynamically stable and do not have complete coronary occlusion
b. Because NSTEMI and UA do not lead to myocardial damage
c. Because PCI is only effective for STEMI patients
d. Because thrombolytic therapy is a better option for these patients
a. Because they are usually hemodynamically stable and do not have complete coronary occlusion
Rationale: Unlike STEMI, which involves total coronary occlusion, UA and NSTEMI are typically caused by partial or transient thrombosis. This allows time for cardiac catheterization within 12 to 72 hours rather than requiring emergent intervention.
What is the primary goal of initial management in a patient presenting with acute coronary syndrome (ACS)?
a. Reduce blood pressure to normal levels
b. Preserve heart muscle and quickly diagnose and treat ACS
c. Perform immediate cardiac catheterization in all cases
d. Administer thrombolytic therapy to all patients
b. Preserve heart muscle and quickly diagnose and treat ACS
Rationale: Rapid diagnosis and treatment are crucial in ACS to minimize myocardial damage and improve patient outcomes.
What is the recommended position for a patient with ACS upon arrival at the emergency department?
a. Supine, unless contraindicated
b. Left lateral, unless contraindicated
c. Trendelenburg, unless contraindicated
d. Upright, unless contraindicated
d. Upright, unless contraindicated
Rationale: Positioning the patient upright improves oxygenation and reduces cardiac workload.
Which medication should be administered first for a patient with chest pain suspected to be ACS, if not already given before arrival?
a. IV heparin
b. Chewable aspirin (162-325 mg)
c. IV morphine
d. High-dose statin
c. IV b. Chewable aspirin (162-325 mg)
Rationale: Aspirin is given as soon as possible to inhibit platelet aggregation and reduce infarct size.
What is the purpose of administering sublingual nitroglycerin (SL NTG) to a patient with ACS?
a. To lower blood pressure rapidly
b. To relieve chest pain by dilating coronary arteries
c. To prevent clot formation
d. To decrease heart rate
b. To relieve chest pain by dilating coronary arteries
Rationale: NTG improves blood flow to the ischemic myocardium by dilating coronary arteries.
What laboratory tests should be obtained upon initial evaluation of a patient with ACS?
a. Complete blood count only
b. Blood glucose and renal function tests only
c. Cardiac biomarkers, complete blood count, and coagulation studies
d. D-dimer and liver function tests
c. Cardiac biomarkers, complete blood count, and coagulation studies
Rationale: Cardiac biomarkers confirm myocardial injury, and coagulation studies guide anticoagulation therapy.
When should morphine be administered to a patient with ACS?
a. Only if pain is unrelieved by nitroglycerin
b. Immediately upon arrival at the emergency department
c. Before administering aspirin
d. Only in patients with ST elevation
a. Only if pain is unrelieved by nitroglycerin
Rationale: Morphine is a second-line agent for pain relief when nitroglycerin fails to alleviate symptoms.
A patient with ACS has an ECG showing ST elevation. What is the immediate next step?
a. Admit the patient to the telemetry unit
b. Start heparin and GP IIb/IIIa inhibitors
c. Send the patient for immediate cardiac catheterization
d. Administer serial cardiac biomarkers before proceeding
c. Send the patient for immediate cardiac catheterization
Rationale: In STEMI, immediate PCI is the preferred intervention to restore coronary perfusion.
In a facility without PCI capability, what is the recommended treatment for a patient with STEMI?
a. Transfer the patient within 6 hours
b. Initiate thrombolytic therapy within 30 minutes
c. Administer heparin and wait for biomarker results
d. Perform bedside percutaneous coronary intervention
b. Initiate thrombolytic therapy within 30 minutes
Rationale: If PCI is unavailable, thrombolytic therapy is started within 30 minutes to dissolve the thrombus.
How are dysrhythmias managed in patients with ACS?
a. Treat according to agency protocols
b. Start all patients on amiodarone
c. Only treat if life-threatening
d. Administer magnesium sulfate in all cases
a. Treat according to agency protocols
Rationale: Dysrhythmias are common in ACS and must be managed based on their type and severity following hospital protocols.
What is the role of glycoprotein (GP) IIb/IIIa inhibitors in ACS management?
a. Prevent platelet aggregation in patients undergoing PCI
b. Reduce blood pressure in hypertensive patients
c. Dissolve existing thrombi in STEMI patients
d. Decrease myocardial oxygen demand
a. Prevent platelet aggregation in patients undergoing PCI
Rationale: GP IIb/IIIa inhibitors, such as eptifibatide, are potent antiplatelet agents used in PCI to reduce thrombotic complications.
A patient is diagnosed with NSTEMI. What is the recommended timeline for cardiac catheterization in this patient?
a. Within 30 minutes
b. Within 12 to 72 hours
c. Within 6 hours
d. Only if medical therapy fails
b. Within 12 to 72 hours
Rationale: NSTEMI patients require catheterization within 12 to 72 hours to evaluate the extent of coronary artery disease and determine treatment.
Why is a high-dose statin (e.g., atorvastatin 80 mg) given to patients with ACS?
a. To stabilize plaque and reduce future cardiovascular events
b. To increase HDL cholesterol
c. To lower blood pressure
d. To immediately dissolve thrombi
a. To stabilize plaque and reduce future cardiovascular events
Rationale: High-dose statins are given to ACS patients to reduce LDL cholesterol and stabilize atherosclerotic plaques, lowering the risk of recurrent events.
What is the primary goal of emergent percutaneous coronary intervention (PCI) in a patient with STEMI?
a. Reduce blood pressure and heart rate
b. Open the blocked artery within 90 minutes of arrival
c. Prevent the need for coronary artery bypass grafting (CABG)
d. Avoid the use of thrombolytic therapy
b. Open the blocked artery within 90 minutes of arrival
Rationale: Emergent PCI aims to rapidly restore blood flow to the infarcted area, limiting myocardial damage and improving patient outcomes.
What is the primary advantage of PCI over CABG in the treatment of STEMI?
a. PCI has a lower risk of restenosis
b. PCI is a more definitive treatment for multivessel disease
c. PCI provides faster reperfusion and requires less recovery time
d. PCI does not require antiplatelet therapy after the procedure
c. PCI provides faster reperfusion and requires less recovery time
Rationale: PCI is preferred over CABG for STEMI due to its ability to rapidly restore perfusion with shorter hospital stays and recovery times.
What assessment is performed during cardiac catheterization for a STEMI patient?
a. Measurement of ejection fraction
b. Myocardial biopsy
c. Cardiac troponin levels
d. Endomyocardial mapping
a. Measurement of ejection fraction
Rationale: During catheterization, left ventricular (LV) function is assessed to determine the impact of the infarction and overall cardiac performance.
What is the purpose of drug-eluting stents (DES) in PCI?
a. To release medication that prevents restenosis
b. To prevent blood clots from forming in the heart
c. To dissolve thrombi within the coronary artery
d. To increase the size of collateral circulation
a. To release medication that prevents restenosis
Rationale: DES release antiproliferative drugs that reduce the risk of re-narrowing (restenosis) of the stented artery.
When is PCI performed on non-infarct-related coronary arteries during an emergent catheterization for STEMI?
a. Only if the patient is hemodynamically stable
b. Only if the blockages exceed 90% stenosis
c. Always at the same time as the primary PCI
d. Never, as it increases mortality risk
a. Only if the patient is hemodynamically stable
Rationale: If the patient is stable, PCI may be performed on additional blockages. Otherwise, elective PCI is scheduled later.
What long-term antiplatelet therapy is required after PCI for acute coronary syndrome (ACS)?
a. Warfarin for 3 months, followed by aspirin for life
b. Dual antiplatelet therapy (DAPT) for 1 year, then aspirin for life
c. Clopidogrel indefinitely
d. Heparin infusion for 6 months
b. Dual antiplatelet therapy (DAPT) for 1 year, then aspirin for life
Rationale: DAPT (aspirin plus a P2Y12 inhibitor like ticagrelor or prasugrel) reduces stent thrombosis risk. Aspirin is continued lifelong.
Which patients may require ventricular assist device (VAD) therapy after PCI?
a. Patients with normal left ventricular function
b. Patients with single-vessel coronary artery disease
c. Patients with no symptoms after PCI
d. Patients with severe left ventricular dysfunction
d. Patients with severe left ventricular dysfunction
Rationale: Patients with significant LV dysfunction may need VAD therapy for circulatory support to prevent heart failure progression.
Which complication of PCI could lead to an emergent CABG?
a. Chronic stable angina
b. Coronary artery dissection or rupture
c. Mild restenosis of the stented artery
d. Low ejection fraction
b. Coronary artery dissection or rupture
Rationale: Coronary artery dissection, rupture, or stent failure may require immediate surgical intervention via CABG.
What is a potential risk during PCI that could worsen a myocardial infarction?
a. Hypertension due to sedative medications
b. Reduction in myocardial oxygen demand
c. Decreased need for long-term anticoagulation
d. Dislodgement of atherosclerotic plaque, causing distal embolization
d. Dislodgement of atherosclerotic plaque, causing distal embolization
Rationale: Plaque disruption during PCI may occlude distal vessels, worsening infarction.
How long is the typical hospital stay for a patient after an uncomplicated PCI for MI?
a. 1 day
b. 3 to 4 days
c. 6 to 8 days
d. 10 days
b. 3 to 4 days
Rationale: Patients undergoing PCI for an uncomplicated MI typically stay in the hospital for 3 to 4 days, which is shorter than CABG patients.
What is a major advantage of PCI compared to thrombolytic therapy in STEMI management?
a. PCI eliminates the need for dual antiplatelet therapy
b. PCI has a lower risk of intracranial hemorrhage
c. PCI is available in all hospitals
d. PCI is effective even if delayed for 12 hours
b. PCI has a lower risk of intracranial hemorrhage
Rationale: PCI is preferred over thrombolytics due to its reduced risk of major bleeding, including intracranial hemorrhage.
Why do some STEMI patients with additional coronary artery blockages return later for elective PCI instead of having all lesions treated during the initial emergent procedure?
a. To minimize contrast-induced nephropathy and hemodynamic instability
b. To avoid stent thrombosis
c. Because PCI can only be performed once per patient
d. Because additional PCI is only beneficial in asymptomatic patients
a. To minimize contrast-induced nephropathy and hemodynamic instability
Rationale: Performing multiple PCIs at once increases contrast exposure and procedural risks. Elective PCI allows for safer staged treatment.
Which patient is the best candidate for thrombolytic therapy?
a. A patient with NSTEMI and elevated troponin levels
b. A patient with STEMI who can be transferred to a PCI-capable hospital within 90 minutes
c. A patient with STEMI in a hospital without PCI capability, arriving 20 minutes ago
d. A patient with chronic stable angina and ST-segment depression
c. A patient with STEMI in a hospital without PCI capability, arriving 20 minutes ago
Rationale: Thrombolytic therapy is only indicated for STEMI patients when PCI is unavailable or cannot be performed within 120 minutes. The goal is to administer the thrombolytic within 30 minutes of arrival.
What is the primary goal of thrombolytic therapy in STEMI management?
a. Increase collateral circulation and restore coronary artery perfusion
b. Reduce myocardial oxygen demand and restore coronary artery perfusion
c. Dissolve the thrombus and restore coronary artery perfusion
d. Strengthen the heart muscle after infarction and restore coronary artery perfusion
c. Dissolve the thrombus and restore coronary artery perfusion
Rationale: Thrombolytic therapy works by breaking down the fibrin in the clot, restoring blood flow to the myocardium and limiting infarct size.
When should thrombolytic therapy be administered to a patient with STEMI who is not in a PCI-capable hospital?
a. Within 15 minutes of arrival
b. Within 30 minutes of arrival
c. Within 60 minutes of symptom onset
d. Within 90 minutes of arrival
b. Within 30 minutes of arrival
Rationale: The guideline-recommended goal is to administer thrombolytics within 30 minutes of ED arrival to optimize myocardial salvage.
A patient with STEMI presents to a non–PCI-capable hospital. What is the best course of action if the patient can be transferred to a PCI-capable hospital within 120 minutes?
a. Give thrombolytics immediately before transfer
b. Administer thrombolytics and then transfer
c. Observe the patient for 30 minutes before deciding on treatment
d. Transfer the patient for PCI without thrombolytics
d. Transfer the patient for PCI without thrombolytics
Rationale: If PCI can be performed within 120 minutes of first medical contact, direct transfer is preferred over thrombolytic therapy because PCI is more effective in restoring coronary perfusion and reducing complications.
What is a major contraindication to thrombolytic therapy in a patient with STEMI?
a. Hypertension with a BP of 150/90 mmHg
b. History of gastrointestinal bleeding 8 months ago
c. Recent ischemic stroke within the past month
d. Age over 75 years
c. Recent ischemic stroke within the past month
Rationale: A recent ischemic stroke is a contraindication due to the high risk of intracranial hemorrhage with thrombolytic therapy.
Why is PCI preferred over thrombolytic therapy when available?
a. PCI restores coronary perfusion more effectively with a lower risk of bleeding
b. PCI requires no antiplatelet or anticoagulant therapy afterward
c. PCI is the only treatment that can limit infarct size
d. PCI can be performed in all hospitals
a. PCI restores coronary perfusion more effectively with a lower risk of bleeding
Rationale: PCI is more effective than thrombolytics in reestablishing blood flow and has a lower risk of major bleeding, particularly intracranial hemorrhage.
Which patient is the best candidate for thrombolytic therapy?
a. A patient with NSTEMI and elevated troponin levels
b. A patient with STEMI, chest pain for 10 hours, and no contraindications
c. A patient with STEMI, chest pain for 14 hours, and a history of hemorrhagic stroke
d. A patient with chest pain for 8 hours but no ST-segment elevation on ECG
b. A patient with STEMI, chest pain for 10 hours, and no contraindications
Rationale: Thrombolytic therapy is indicated for STEMI patients who present within 12 hours of symptom onset and have no absolute contraindications.
What is the primary mechanism of action of thrombolytic agents in STEMI management?
a. Stabilizing platelet aggregation
b. Inhibiting thrombin formation
c. Lysing the fibrin mesh of the thrombus
d. Reducing myocardial oxygen demand
c. Lysing the fibrin mesh of the thrombus
Rationale: Thrombolytic agents break down fibrin in the thrombus, restoring blood flow to the occluded coronary artery.
A patient is receiving thrombolytic therapy for STEMI. Which assessment finding requires immediate intervention?
a. A brief episode of mild gingival bleeding
b. A sudden decrease in level of consciousness
c. A decrease in ST-segment elevation on ECG
d. A blood pressure of 138/86 mmHg
b. A sudden decrease in level of consciousness
Rationale: A sudden change in neurological status may indicate intracranial hemorrhage, a severe complication of thrombolytic therapy.
A patient presents with STEMI and chest pain that began 13 hours ago. What is the best course of action?
a. Evaluate the patient for possible PCI
b. Administer thrombolytic therapy immediately
c. Start IV heparin and monitor for symptom progression
d. Perform an echocardiogram before determining treatment
a. Evaluate the patient for possible PCI
Rationale: Thrombolytics are most effective within 12 hours of symptom onset. If chest pain has lasted 12-24 hours, PCI is preferred when available.
A patient with STEMI is being evaluated for thrombolytic therapy. Which condition is an absolute contraindication?
a. A history of gastrointestinal bleeding 1 year ago
b. Ischemic stroke 2 months ago
c. BP of 160/100 mmHg
d. Active internal bleeding
d. Active internal bleeding
Rationale: Active internal bleeding is an absolute contraindication due to the risk of life-threatening hemorrhage.
Which statement about thrombolytic therapy in STEMI patients is correct?
a. All STEMI patients should receive thrombolytics
b. Thrombolytics should be given only in PCI-capable hospitals
c. Thrombolytics can be considered for some patients with symptom onset 12-24 hours ago
d. Patients must have a history of prior MI to qualify for thrombolytic therapy
c. Thrombolytics can be considered for some patients with symptom onset 12-24 hours ago
Rationale: While thrombolytics are most effective within 12 hours, some patients with persistent ischemic symptoms and ECG changes may still benefit within 12-24 hours.
Before administering thrombolytic therapy, which action should the nurse take to reduce the risk of bleeding complications?
a. Start one large-bore IV line
b. Administer heparin before the thrombolytic
c. Perform all invasive procedures before giving the thrombolytic
d. Ensure the patient has a blood pressure below 120/80 mmHg
c. Perform all invasive procedures before giving the thrombolytic
Rationale: Performing invasive procedures before administration reduces the risk of significant bleeding once the thrombolytic agent is given.
Which clinical finding is the most reliable indicator of successful reperfusion after thrombolytic therapy?
a. Resolution of chest pain
b. Return of ST segment to baseline on ECG
c. Development of premature ventricular contractions (PVCs)
d. A decrease in cardiac biomarker levels
b. Return of ST segment to baseline on ECG
Rationale: The most reliable sign of reperfusion is normalization of the ST segment, indicating restoration of blood flow.
A patient receiving thrombolytic therapy for STEMI reports new chest pain. What is the priority nursing action?
a. Reassess the ECG for ST-segment changes
b. Increase the IV fluid rate
c. Discontinue the thrombolytic infusion immediately
d. Administer an additional dose of thrombolytic agent
a. Reassess the ECG for ST-segment changes
Rationale: Recurring chest pain and ST-segment elevation may indicate reocclusion, requiring reassessment and possible PCI.
A patient receiving thrombolytic therapy develops altered mental status and a severe headache. What is the priority action?
a. Continue the infusion but lower the rate
b. Assess blood glucose levels
c. Discontinue the thrombolytic infusion immediately
d. Administer IV fluids to increase perfusion
c. Discontinue the thrombolytic infusion immediately
Rationale: Neurological changes may indicate cerebral hemorrhage, a life-threatening complication of thrombolytic therapy.
What is an expected finding in a patient undergoing thrombolytic therapy?
a. A rapid drop in cardiac biomarker levels
b. A decrease in oxygen saturation below 90%
c. An increase in systolic blood pressure above 180 mmHg
d. Development of minor surface bleeding at IV sites
d. Development of minor surface bleeding at IV sites
Rationale: Minor bleeding, such as at IV sites or the gums, is common and can often be managed with pressure or ice packs.
What is the primary purpose of administering heparin following thrombolytic therapy?
a. To prevent reocclusion of the coronary artery
b. To dissolve residual thrombus
c. To manage reperfusion dysrhythmias
d. To reverse the effects of the thrombolytic agent
a. To prevent reocclusion of the coronary artery
Rationale: Heparin helps maintain vessel patency and prevents new clot formation after thrombolysis.
A patient with STEMI received thrombolytic therapy 45 minutes ago. Which finding requires immediate intervention?
a. BP 130/85 mmHg
b. Accelerated idioventricular rhythm on ECG
c. Oozing at the IV insertion site
d. A sudden drop in hemoglobin and hematocrit
d. A sudden drop in hemoglobin and hematocrit
Rationale: A drop in hemoglobin and hematocrit suggests significant internal bleeding, a serious complication of thrombolytic therapy.
A nurse is monitoring a patient after thrombolytic therapy. Which reperfusion dysrhythmia is most commonly seen and does not typically require treatment?
a. Atrial fibrillation
b. Ventricular tachycardia
c. Accelerated idioventricular rhythm
d. Complete heart block
c. Accelerated idioventricular rhythm
Rationale: Accelerated idioventricular rhythm is a common reperfusion dysrhythmia that is usually self-limiting and does not require aggressive intervention.
After thrombolytic therapy, a patient is scheduled for PCI at a facility with interventional cardiology. What is the primary reason for transferring the patient?
a. To monitor for recurrent chest pain
b. To perform a coronary angiogram and possible stent placement
c. To assess the patient’s response to thrombolytic therapy
d. To provide extended cardiac rehabilitation services
b. To perform a coronary angiogram and possible stent placement
Rationale: Patients receiving thrombolytics should be transferred to a PCI-capable facility to undergo angiography and possible PCI if needed.
A patient receiving thrombolytic therapy has persistent hypotension and tachycardia. What is the nurse’s priority concern?
a. Cardiogenic shock
b. Fluid overload
c. Pulmonary embolism
d. Hemorrhage
d. Hemorrhage
Rationale: Persistent hypotension and tachycardia suggest significant blood loss, which is a major complication of thrombolytic therapy.
Which patient receiving thrombolytic therapy for STEMI is at highest risk for a life-threatening complication?
a. A 72-year-old with a history of peptic ulcer disease
b. A 60-year-old with a BP of 145/92 mmHg
c. A 55-year-old with chronic kidney disease
d. A 50-year-old with a history of ischemic stroke 1 month ago
d. A 50-year-old with a history of ischemic stroke 1 month ago
Rationale: A recent ischemic stroke is an absolute contraindication for thrombolytic therapy due to the high risk of intracranial hemorrhage.
A patient with suspected acute coronary syndrome (ACS) arrives at the emergency department. Which medication should be administered first?
a. IV heparin
b. Oral β-blocker
c. Chewable aspirin
d. IV nitroglycerin
c. Chewable aspirin
Rationale: Chewable aspirin should be given immediately to inhibit platelet aggregation and reduce myocardial damage.
A patient with STEMI is scheduled for PCI. Which drug class may be administered during the procedure to prevent platelet aggregation?
a. Calcium channel blockers
b. GP IIb/IIIa inhibitors
c. Low-molecular-weight heparin (LMWH)
d. High-dose atorvastatin
b. GP IIb/IIIa inhibitors
Rationale: GP IIb/IIIa inhibitors prevent platelet aggregation and are often used during PCI in STEMI patients.
A patient with NSTEMI is receiving medical management without immediate PCI. What anticoagulation therapy is recommended?
a. IV nitroglycerin
b. Subcutaneous LMWH or IV unfractionated heparin (UH)
c. High-dose aspirin alone
d. GP IIb/IIIa inhibitors only
b. Subcutaneous LMWH or IV unfractionated heparin (UH)
Rationale: Systemic anticoagulation with LMWH or UH is recommended for NSTEMI patients managed without immediate PCI.
A patient is started on DAPT after experiencing a STEMI. How long should DAPT be continued?
a. 1 month
b. 3 months
c. 6 months
d. 1 year
d. 1 year
Rationale: DAPT should be continued for at least 1 year after STEMI or NSTEMI, regardless of stent placement.
A patient with ACS has severe hypotension. Which medication should be avoided?
a. Atorvastatin
b. Aspirin
c. IV nitroglycerin
d. β-blockers
d. β-blockers
Rationale: β-blockers lower blood pressure and should not be given to patients with severe hypotension.
A patient with ACS is unable to tolerate β-blockers due to severe asthma. What medication can be used instead?
a. ACE inhibitors
b. Calcium channel blockers
c. GP IIb/IIIa inhibitors
d. IV nitroglycerin
b. Calcium channel blockers
Rationale: Calcium channel blockers are used when β-blockers are contraindicated, though they must be used cautiously due to their effects on contractility.
A patient with STEMI is prescribed atorvastatin 80 mg. What is the primary goal of this therapy?
a. Reduce chest pain
b. Prevent further clot formation
c. Lower LDL cholesterol and stabilize plaque
d. Improve myocardial contractility
c. Lower LDL cholesterol and stabilize plaque
Rationale: High-dose atorvastatin lowers LDL and stabilizes atherosclerotic plaques, reducing the risk of further cardiac events.
Which patient should NOT receive ACE inhibitor therapy after a myocardial infarction (MI)?
a. A patient with an EF of 35%
b. A patient with chronic kidney disease and hyperkalemia
c. A patient with a history of hypertension
d. A patient with diabetes mellitus
b. A patient with chronic kidney disease and hyperkalemia
Rationale: ACE inhibitors can increase potassium levels and worsen kidney function, making them contraindicated in patients with hyperkalemia.
A patient with persistent chest pain after a STEMI is prescribed nitrates. What is an important consideration?
a. Nitrates should be taken only in the morning
b. Nitrates should be avoided in patients taking phosphodiesterase inhibitors (e.g., sildenafil)
c. Nitrates should replace DAPT in long-term therapy
d. Nitrates increase blood pressure and should be given cautiously
b. Nitrates should be avoided in patients taking phosphodiesterase inhibitors (e.g., sildenafil)
Rationale: The combination of nitrates and phosphodiesterase inhibitors can cause severe hypotension.
A patient with NSTEMI is receiving ticagrelor as part of DAPT. What patient education should the nurse provide?
a. “You can stop taking aspirin after 6 months.”
b. “Take this medication on an empty stomach for best absorption.”
c. “You may take this medication only on days when you have chest pain.”
d. “Avoid grapefruit juice while taking this medication.”
d. “Avoid grapefruit juice while taking this medication.”
Rationale: Grapefruit juice can interfere with ticagrelor metabolism, increasing the risk of bleeding.
A patient with ACS is receiving IV nitroglycerin. What is the primary goal of this therapy?
a. Increase blood pressure to improve perfusion
b. Decrease preload and afterload while increasing myocardial oxygen supply
c. Increase heart rate to enhance cardiac output
d. Prevent clot formation in the coronary arteries
b. Decrease preload and afterload while increasing myocardial oxygen supply
Rationale: IV nitroglycerin reduces preload and afterload, decreasing myocardial workload while increasing oxygen delivery to the heart.
A patient receiving IV nitroglycerin reports dizziness and lightheadedness. The nurse assesses a BP of 88/56 mmHg. What is the priority intervention?
a. Increase the nitroglycerin infusion rate
b. Discontinue the infusion immediately
c. Administer an IV fluid bolus
d. Encourage the patient to ambulate to improve circulation
c. Administer an IV fluid bolus
Rationale: Hypotension is a common side effect of IV nitroglycerin, and volume depletion may contribute to it. A fluid bolus helps restore intravascular volume.
A patient with ACS is started on IV nitroglycerin but continues to have chest pain. What is the appropriate nursing action?
a. Titrate the IV nitroglycerin to relieve chest pain
b. Discontinue the IV nitroglycerin and switch to sublingual NTG
c. Administer an IV beta-blocker immediately
d. Increase the patient’s fluid intake to prevent hypotension
a. Titrate the IV nitroglycerin to relieve chest pain
Rationale: IV nitroglycerin should be titrated based on pain relief while monitoring blood pressure closely.
The nurse is monitoring a patient receiving IV nitroglycerin. Which finding requires immediate intervention?
a. BP of 100/60 mmHg
b. Development of crackles in the lungs
c. Mild headache
d. Heart rate of 78 bpm
b. Development of crackles in the lungs
Rationale: Crackles indicate possible volume overload, which can occur if the patient received a fluid bolus for hypotension. Immediate assessment and intervention are needed.
A patient with ACS is receiving IV nitroglycerin. Which statement by the patient requires further assessment?
a. “My chest pain is getting worse.”
b. “I feel a mild headache.”
c. “I feel a little lightheaded when I sit up.”
d. “I feel a warm sensation at the IV site.”
a. “My chest pain is getting worse.”
Rationale: Worsening chest pain may indicate treatment failure or ongoing ischemia, requiring further evaluation.
A nurse is educating a student about IV nitroglycerin use in ACS. Which statement by the student indicates the need for further teaching?
a. “IV nitroglycerin should be titrated based on chest pain relief.”
b. “Patients receiving IV nitroglycerin should be closely monitored for hypotension.”
c. “A fluid bolus may be needed if the patient becomes hypotensive.”
d. “IV nitroglycerin is primarily used to lower blood pressure in ACS patients.”
d. “IV nitroglycerin is primarily used to lower blood pressure in ACS patients.”
Rationale: The primary goal of IV nitroglycerin in ACS is to relieve chest pain and improve coronary blood flow, not just to lower BP.
A nurse is caring for a patient on IV nitroglycerin who develops a systolic BP of 85 mmHg. The nurse should anticipate which order from the provider?
a. Administer a fluid bolus
b. Increase the nitroglycerin infusion rate
c. Start an IV beta-blocker
d. Encourage the patient to deep breathe
a. Administer a fluid bolus
Rationale: Hypotension is a known side effect of IV nitroglycerin, and a fluid bolus can help restore blood pressure if volume depletion is present.
Which patient receiving IV nitroglycerin requires the most immediate intervention?
a. A patient with a headache rating of 4/10
b. A patient with a BP of 98/60 mmHg and no symptoms
c. A patient with new-onset crackles in the lungs
d. A patient with mild dizziness that resolves when lying down
c. A patient with new-onset crackles in the lungs
Rationale: Crackles suggest fluid overload, which requires immediate intervention to prevent worsening heart failure or pulmonary edema.
A patient with ACS continues to have chest pain despite receiving IV nitroglycerin. The provider orders IV morphine. What is the primary reason for administering morphine in this patient?
a. To increase myocardial contractility
b. To lower myocardial oxygen consumption and relieve pain
c. To increase blood pressure and improve perfusion
d. To treat anxiety and promote slee
b. To lower myocardial oxygen consumption and relieve pain
Rationale: Morphine is a vasodilator that decreases myocardial oxygen demand by reducing cardiac workload, contractility, blood pressure, and heart rate, thereby relieving chest pain.
The nurse is monitoring a patient who received IV morphine for chest pain. Which finding requires immediate intervention?
a. Blood pressure of 98/60 mmHg
b. Heart rate of 70 bpm
c. Patient reports mild nausea
d. Respiratory rate of 8 breaths per minute
d. Respiratory rate of 8 breaths per minute
Rationale: Morphine can cause respiratory depression. A respiratory rate below 10 breaths per minute requires immediate intervention, such as stopping the medication and administering naloxone if needed.
A patient with STEMI is given IV morphine for pain relief. Which additional benefit does morphine provide in this setting?
a. It increases myocardial oxygen supply
b. It increases preload and stroke volume
c. It reduces anxiety and sympathetic stimulation
d. It enhances cardiac contractility
c. It reduces anxiety and sympathetic stimulation
Rationale: Morphine helps relieve anxiety and reduces sympathetic nervous system activation, which decreases myocardial oxygen demand and workload.
The nurse is preparing to administer morphine to a patient with ACS. Which assessment finding would require the nurse to hold the dose and contact the provider?
a. Patient rates chest pain as 8/10
b. Blood pressure is 88/54 mmHg
c. Oxygen saturation is 96% on room air
d. Patient reports nausea and dizziness
b. Blood pressure is 88/54 mmHg
Rationale: Morphine is a vasodilator that can further lower blood pressure. If the patient is already hypotensive, administering morphine could worsen perfusion and cause hemodynamic instability.
A nurse is teaching a student about morphine use in ACS. Which statement by the student indicates the need for further education?
a. “Morphine can help relieve chest pain when nitroglycerin is ineffective.”
b. “Morphine should be used cautiously because it can depress respirations.”
c. “Morphine is the first-line drug for pain relief in all ACS patients.”
d. “Morphine lowers myocardial oxygen demand by reducing cardiac workload.”
c. “Morphine is the first-line drug for pain relief in all ACS patients.”
Rationale: Nitroglycerin is the first-line medication for chest pain in ACS. Morphine is used only when chest pain persists despite nitroglycerin.
A patient with ACS who received IV morphine becomes drowsy and has a respiratory rate of 10 breaths per minute. What is the nurse’s priority action?
a. Place the patient in high Fowler’s position
b. Administer naloxone as prescribed
c. Encourage the patient to take deep breaths
d. Decrease the oxygen flow rate
b. Administer naloxone as prescribed
Rationale: Naloxone is an opioid antagonist that reverses the effects of morphine-induced respiratory depression. If the patient’s respiratory status declines further, naloxone should be administered immediately.
A patient with ACS is prescribed a β-blocker. Which finding would require the nurse to hold the medication and notify the provider?
a. Heart rate of 48 bpm
b. Blood pressure of 130/80 mmHg
c. Patient reports mild fatigue
d. Respiratory rate of 18 breaths per minute
a. Heart rate of 48 bpm
Rationale: β-blockers reduce heart rate, blood pressure, and myocardial oxygen demand. A heart rate below 50 bpm may indicate excessive β-blockade, increasing the risk of bradycardia and hemodynamic instability.
A nurse is teaching a patient about β-blockers after an MI. Which statement by the patient indicates a need for further education?
a. “I will take this medication for the rest of my life.”
b. “This medication helps prevent another heart attack.”
c. “I should stop taking this medication if I feel dizzy.”
d. “I may feel more tired than usual when I start this medication.”
c. “I should stop taking this medication if I feel dizzy.”
Rationale: β-blockers should not be stopped abruptly, as this can cause rebound tachycardia and increase the risk of reinfarction. Instead, the patient should report dizziness to the provider for dose adjustment.
A patient with STEMI is started on a β-blocker. What is the primary reason for prescribing this medication?
a. To increase cardiac output and tissue perfusion
b. To prevent ventricular remodeling and reduce mortality
c. To increase myocardial contractility and stroke volume
d. To decrease coronary artery vasospasm and increase perfusion
b. To prevent ventricular remodeling and reduce mortality
Rationale: β-blockers reduce myocardial oxygen demand, prevent reinfarction, and decrease ventricular remodeling, which lowers the risk of heart failure and improves long-term survival after an MI.
A nurse is reviewing the medication orders for a patient who had an NSTEMI. The provider prescribes an ACE inhibitor. Which patient condition would require the nurse to question this order?
a. Ejection fraction of 35%
b. Blood pressure of 88/56 mmHg
c. History of type 2 diabetes mellitus
d. Serum potassium level of 4.0 mEq/L
b. Blood pressure of 88/56 mmHg
Rationale: ACE inhibitors lower blood pressure, and initiating therapy in a hypotensive patient can worsen hemodynamic instability. Hypotension is a contraindication for starting an ACE inhibitor.
A patient recovering from an STEMI is prescribed an ACE inhibitor. Which statement by the patient indicates a need for further teaching?
a. “This medication will help protect my heart function over time.”
b. “I should report any swelling of my face or lips to my doctor.”
c. “If I develop a dry cough, I should stop taking this medication immediately.”
d. “I should have my kidney function and potassium levels monitored regularly.”
c. “If I develop a dry cough, I should stop taking this medication immediately.”
Rationale: A dry cough is a common side effect of ACE inhibitors, but the patient should not stop the medication abruptly without consulting a provider. The provider may switch the patient to an ARB if the cough is bothersome.
A patient with a history of heart failure and a recent STEMI is prescribed an ARB instead of an ACE inhibitor. What is the most likely reason for this substitution?
a. The patient has hypertension
b. The patient has diabetes mellitus
c. The patient experienced angioedema with an ACE inhibitor
d. The patient prefers once-daily dosing
c. The patient experienced angioedema with an ACE inhibitor
Rationale: ARBs are used as an alternative to ACE inhibitors for patients who develop angioedema, a potentially life-threatening side effect of ACE inhibitors.
A nurse is monitoring a patient who started an ACE inhibitor after an NSTEMI. Which finding is most concerning and requires immediate intervention?
a. Persistent dry cough
b. Blood pressure of 118/72 mmHg
c. Mild dizziness when standing
d. Serum potassium level of 5.8 mEq/L
d. Serum potassium level of 5.8 mEq/L
Rationale: ACE inhibitors can cause hyperkalemia by decreasing aldosterone secretion. A potassium level of 5.8 mEq/L is above the normal range and increases the risk of cardiac arrhythmias.
A patient asks why they need to continue taking an ACE inhibitor indefinitely after an NSTEMI. What is the nurse’s best response?
a. “ACE inhibitors help control your blood pressure.”
b. “This medication prevents fluid buildup in your lungs.”
c. “ACE inhibitors reduce the workload on your heart and prevent heart failure.”
d. “This medication prevents blood clots from forming in your arteries.”
c. “ACE inhibitors reduce the workload on your heart and prevent heart failure.”
Rationale: ACE inhibitors prevent ventricular remodeling and slow the progression of heart failure by reducing afterload and preload, which helps preserve cardiac function long-term.
The nurse is preparing to administer the first dose of an ACE inhibitor to a post-MI patient. What should the nurse do first?
a. Monitor the patient’s blood pressure
b. Assess the patient’s bowel sounds
c. Check the patient’s blood glucose level
d. Evaluate the patient’s capillary refill
a. Monitor the patient’s blood pressure
Rationale: Hypotension is a potential adverse effect of ACE inhibitors, especially with the first dose. The nurse should assess BP before administration to ensure it is within a safe range.
A patient with an NSTEMI is started on an ACE inhibitor. What additional teaching should the nurse provide regarding this medication?
a. “Avoid potassium-containing salt substitutes.”
b. “Increase your intake of green leafy vegetables.”
c. “Take this medication at bedtime to avoid dizziness.”
d. “Expect to see an improvement in chest pain within 24 hours.”
a. “Avoid potassium-containing salt substitutes.”
Rationale: ACE inhibitors can cause hyperkalemia, and potassium-containing salt substitutes may further increase potassium levels, leading to dangerous arrhythmias.
A nurse is caring for a patient who has developed a dysrhythmia following a recent myocardial infarction (MI). The patient is asymptomatic, and the dysrhythmia is self-limiting. Which of the following interventions is most appropriate?
a. Administer an antidysrhythmic medication to convert the rhythm.
b. Monitor the patient’s rhythm continuously and assess for changes in symptoms.
c. Prepare the patient for immediate cardioversion.
d. Begin administering intravenous fluids to restore electrolyte balance.
b. Monitor the patient’s rhythm continuously and assess for changes in symptoms.
Rationale: Since most dysrhythmias after MI are self-limiting and not life-threatening, the appropriate intervention is to monitor the patient closely for any changes in symptoms and rhythm, rather than aggressively treating it with antidysrhythmic medications or other interventions.
A patient with a recent myocardial infarction (MI) develops a sustained ventricular tachycardia (VT). The nurse knows that this type of dysrhythmia is potentially life-threatening. Which of the following actions should the nurse prioritize?
a. Prepare to administer an antidysrhythmic drug such as amiodarone.
b. Observe the patient’s rhythm and continue monitoring vital signs.
c. Notify the healthcare provider and prepare for possible defibrillation.
d. Administer oxygen and prepare for cardioversion.
c. Notify the healthcare provider and prepare for possible defibrillation.
Rationale: Sustained VT is a life-threatening dysrhythmia that may require defibrillation if it does not respond to medical management. The nurse should prioritize notifying the healthcare provider and preparing for possible defibrillation to manage the rhythm effectively.
A nurse is caring for a patient with acute coronary syndrome (ACS). The healthcare provider orders a lipid-lowering drug for the patient. Which of the following is the most important consideration when initiating this therapy?
a. The patient’s cholesterol levels prior to admission.
b. Whether the patient is experiencing symptoms of heart failure.
c. Maximizing the dose of the current lipid-lowering medication or switching to a more potent statin.
d. Assessing the patient’s ability to exercise.
c. Maximizing the dose of the current lipid-lowering medication or switching to a more potent statin.
Rationale: For patients with ACS or coronary artery disease (CAD), the primary goal is to initiate or optimize lipid-lowering therapy to reduce cardiovascular risks. Maximizing the dose of the current medication or switching to a more potent statin is essential in these cases.
A patient with acute coronary syndrome (ACS) is prescribed a lipid-lowering medication. The nurse is aware that which of the following factors should be considered when prescribing this medication?
a. The patient’s cholesterol levels before hospitalization.
b. The presence of renal failure or liver dysfunction.
c. The patient’s exercise tolerance.
d. The patient’s age and gender.
b. The presence of renal failure or liver dysfunction.
Rationale: When prescribing lipid-lowering drugs, especially statins, the nurse should consider any contraindications such as renal failure or liver dysfunction. These conditions can affect the metabolism of statins and increase the risk of adverse effects.
A nurse is educating a patient with acute coronary syndrome (ACS) who has been prescribed a statin. The nurse should emphasize that the patient will need to continue taking the lipid-lowering drug indefinitely unless which of the following occurs?
a. The patient experiences muscle pain or weakness.
b. The patient has an allergic reaction to the medication.
c. The patient’s cholesterol levels return to normal.
d. The patient is no longer experiencing chest pain.
a. The patient experiences muscle pain or weakness.
Rationale: Lipid-lowering drugs, particularly statins, are generally taken indefinitely in patients with ACS or coronary artery disease (CAD) to reduce the risk of future cardiac events. However, treatment may need to be adjusted or discontinued if the patient experiences muscle pain or weakness, which can be a sign of a rare but serious side effect (myopathy).
A nurse is caring for a patient post-STEMI and the healthcare provider orders an aldosterone antagonist. Which of the following assessments is most important before administering this medication?
a. Serum calcium levels
b. Renal function tests
c. Liver function tests
d. Serum potassium levels
d. Serum potassium levels
Rationale: Aldosterone antagonists, such as spironolactone or eplerenone, can increase potassium levels. Therefore, it is crucial to assess serum potassium levels before administering the medication to avoid the risk of hyperkalemia.
A patient with a STEMI is started on an aldosterone antagonist. Which of the following conditions would be a contraindication to the use of this medication?
a. Hyperkalemia
b. Hypernatremia
c. Hypokalemia
d. Hypertension
a. Hyperkalemia
Rationale: Aldosterone antagonists are contraindicated in patients with hyperkalemia as they can further increase potassium levels, which can be dangerous and lead to cardiac arrhythmias.
A nurse is educating a patient who is prescribed an aldosterone antagonist following a STEMI. The nurse should inform the patient that the drug is intended to:
a. Improve myocardial contractility
b. Decrease the risk of heart failure progression
c. Lower cholesterol levels
d. Increase the heart rate
b. Decrease the risk of heart failure progression
Rationale: Aldosterone antagonists like spironolactone and eplerenone are used after a STEMI to reduce mortality and decrease the risk of heart failure progression, regardless of the left ventricular (LV) function. They work by blocking the effects of aldosterone, which can worsen heart failure.
A patient recovering from a myocardial infarction (MI) is prescribed a stool softener to prevent constipation. What is the primary reason for this intervention?
a. To increase bowel motility and prevent diarrhea
b. To prevent straining and vagal stimulation that can lead to dysrhythmias
c. To reduce abdominal discomfort and bloating
d. To reduce the need for opioids
b. To prevent straining and vagal stimulation that can lead to dysrhythmias
Rationale: Stool softeners are used to prevent constipation and avoid straining, which can trigger vagal stimulation and result in bradycardia or dysrhythmias, especially after an MI.
A nurse is educating a patient recovering from a myocardial infarction (MI) about the use of stool softeners. Which of the following should the nurse include in the teaching?
a. “Stool softeners should be used for immediate relief of constipation.”
b. “Straining during a bowel movement is not a concern while on stool softeners.”
c. “Stool softeners help to avoid the potential complications of straining, such as bradycardia.”
d. “Opioid medications are the best treatment for constipation following an MI.”
c. “Stool softeners help to avoid the potential complications of straining, such as bradycardia.”
Rationale: The purpose of stool softeners after an MI is to prevent straining during bowel movements, which can stimulate the vagus nerve, leading to bradycardia and potential dysrhythmias.
A patient recovering from a myocardial infarction (MI) asks the nurse why they were prescribed docusate sodium. The nurse explains that the medication is used to:
a. Stimulate bowel motility to relieve constipation
b. Prevent straining during bowel movements, which could lead to arrhythmias
c. Provide a laxative effect to clear the bowels
d. Prevent nausea and vomiting associated with opioid use
b. Prevent straining during bowel movements, which could lead to arrhythmias
Rationale: Docusate sodium is a stool softener used to prevent straining during bowel movements, which can trigger vagal stimulation and potentially cause bradycardia and arrhythmias in MI patients.
A patient who has just had a myocardial infarction (MI) is receiving opioid pain management and is experiencing constipation. What should the nurse anticipate to prevent complications associated with constipation?
a. Discontinue opioid medications immediately
b. Recommend increasing fluid intake to promote bowel movements
c. Encourage the patient to perform physical activity to improve bowel motility
d. Administer a stool softener to prevent straining during bowel movements
d. Administer a stool softener to prevent straining during bowel movements
Rationale: Opioids can contribute to constipation, and stool softeners are used to prevent straining, which can trigger vagal stimulation and lead to complications such as bradycardia or dysrhythmias following an MI.
When assessing a patient with acute coronary syndrome (ACS), the nurse should be particularly concerned if the patient has a history of which of the following conditions?
a. Heart failure
b. Gallbladder disease
c. Allergies to food
d. Chronic sinusitis
a. Heart failure
Rationale: A history of heart failure is significant as it increases the risk for complications in ACS, making early detection and management of cardiovascular status essential.
During the nursing assessment of a patient with suspected acute coronary syndrome (ACS), the nurse notes the patient has cool, clammy, pale skin. This finding is most likely due to:
a. Fever from decreased cardiac output
b. Dehydration from decreased cardiac output
c. Hypoperfusion from decreased cardiac output
d. Anxiety from decreased cardiac output
c. Hypoperfusion from decreased cardiac output
Rationale: Cool, clammy, pale skin is often indicative of poor perfusion due to reduced cardiac output, which can occur in ACS as a result of decreased heart function.
A nurse is reviewing the health history of a patient with suspected ACS. Which of the following factors is most concerning and warrants further investigation?
a. History of premature coronary artery disease (CAD) in a family member
b. Occasional indigestion
c. A sedentary lifestyle
d. History of hypertension
a. History of premature coronary artery disease (CAD) in a family member
Rationale: A family history of premature CAD is a strong risk factor for developing ACS, making it an important aspect to investigate further.
During the nursing assessment of a patient with ACS, the patient reports substernal chest pain with radiation to the left arm. This is most indicative of:
a. Gastroesophageal reflux disease (GERD)
b. Pulmonary embolism
c. Costochondritis
d. Acute myocardial infarction (MI)
d. Acute myocardial infarction (MI)
Rationale: Substernal chest pain radiating to the left arm is a classic sign of acute myocardial infarction, which should be immediately investigated.
The nurse notes that a patient with ACS is experiencing symptoms of palpitations, dizziness, and weakness. These findings are most likely related to:
a. Hypotension and decreased cardiac output
b. Anxiety and panic attack
c. Hyperglycemia and decreased cardiac output
d. Neurological impairment and panic attack
a. Hypotension and decreased cardiac output
Rationale: Palpitations, dizziness, and weakness in a patient with ACS are typically related to decreased cardiac output and hypotension due to myocardial ischemia.
A patient with suspected ACS has an abnormal ECG showing ST segment elevation. This finding suggests:
a. Aortic dissection
b. Pulmonary embolism (PE)
c. Acute myocardial infarction (STEMI)
d. Heart failure (HF)
c. Acute myocardial infarction (STEMI)
Rationale: ST segment elevation on an ECG is a hallmark of a STEMI, indicating an acute myocardial infarction due to complete occlusion of a coronary artery.
During the nursing assessment of a patient with ACS, the nurse notes that the patient is anxious, fearful, and distressed. This is most likely due to:
a. Increased sympathetic nervous system activation
b. Side effects of medications
c. History of a panic disorder
d. Chest wall discomfort
a. Increased sympathetic nervous system activation
Rationale: Anxiety, fear, and distress in ACS patients are often related to the body’s response to pain and hypoperfusion, which triggers sympathetic nervous system activation.
In the assessment of a patient with ACS, the nurse auscultates an S3 heart sound. This finding may indicate:
a. Aortic stenosis or heart failure
b. Ventricular dilation or heart failure
c. Pericarditis or heart failure
d. Mitral regurgitation or heart failure
b. Ventricular dilation or heart failure
Rationale: An S3 heart sound can be indicative of ventricular dilation or heart failure, which can occur in ACS due to impaired left ventricular function.
Which of the following objective findings would most likely be seen in a patient with acute coronary syndrome (ACS)?
a. Increased WBC count
b. Decreased serum lipids
c. Normal ECG
d. Decreased serum cardiac biomarkers
a. Increased WBC count
Rationale: An elevated WBC count is commonly seen in ACS due to the inflammatory response to myocardial injury.
A patient with suspected ACS presents with palpitations, dyspnea, and chest pain. Upon assessment, the nurse identifies a pulse deficit and dysrhythmias. This suggests:
a. Cardiac tamponade
b. Pulmonary embolism
c. Renal failure
d. Acute myocardial infarction
d. Acute myocardial infarction
Rationale: A pulse deficit and dysrhythmias, especially ventricular, are common in acute myocardial infarction and can be a sign of arrhythmias associated with myocardial injury.
A patient with acute coronary syndrome (ACS) is experiencing anxiety. The nurse understands that anxiety in this patient is most likely due to:
a. The pain associated with myocardial ischemia
b. Fear of death or loss of control
c. The need for invasive procedures
d. Side effects of medications used in ACS
b. Fear of death or loss of control
Rationale: Anxiety in ACS patients is often related to fear of death or loss of control, as they are aware of the severity of their condition and the potential for life-threatening complications.
A patient with ACS is experiencing impaired cardiac function. Which of the following interventions is most appropriate to manage this issue?
a. Administering a stool softener to prevent constipation
b. Encouraging the patient to engage in light physical activity
c. Administering medications to reduce myocardial oxygen demand
d. Restricting fluid intake to prevent fluid overload
c. Administering medications to reduce myocardial oxygen demand
Rationale: In patients with impaired cardiac function, medications such as β-blockers, nitrates, or ACE inhibitors are often used to reduce myocardial oxygen demand and improve cardiac function.
The nurse develops a care plan for a patient with acute coronary syndrome (ACS) and sets the immediate goal of pain relief. Which of the following interventions is most appropriate to achieve this goal?
a. Administering IV morphine to reduce chest pain and myocardial oxygen demand
b. Encouraging the patient to perform deep breathing exercises to relieve pain
c. Providing a warm compress to the chest to alleviate discomfort
d. Encouraging the patient to ambulate as tolerated to relieve chest pain
a. Administering IV morphine to reduce chest pain and myocardial oxygen demand
Rationale: IV morphine is used for pain relief in ACS patients, as it reduces myocardial oxygen demand by decreasing heart rate, blood pressure, and contractility, which helps alleviate chest pain.
A patient with ACS is hospitalized. The nurse focuses on the overall goal of effective coping with illness-associated anxiety. Which of the following interventions is most likely to help achieve this goal?
a. Administering antianxiety medications as prescribed
b. Providing education about the patient’s condition and treatment plan
c. Encouraging the patient to engage in physical activity to reduce anxiety
d. Limiting family visits to reduce patient stimulation and stress
b. Providing education about the patient’s condition and treatment plan
Rationale: Providing education helps patients understand their condition, which can alleviate anxiety and improve coping by reducing fear and uncertainty.
A nurse is caring for a patient with acute coronary syndrome (ACS) who has just been admitted to the ICU. Which of the following actions should the nurse prioritize during the first few hours after admission?
a. Start continuous ECG monitoring
b. Begin daily assessment of dietary intake
c. Administer anticoagulants as prescribed
d. Prepare the patient for discharge planning
a. Start continuous ECG monitoring
Rationale: Continuous ECG monitoring is critical in the first few hours after admission to assess for any dysrhythmias or complications related to ACS.
A patient with STEMI is undergoing reperfusion therapy. Which of the following is the primary goal of reperfusion therapy for STEMI?
a. To reduce blood pressure
b. To prevent deep vein thrombosis
c. To prevent further increases in cholesterol levels
d. To save as much heart muscle as possible
d. To save as much heart muscle as possible
Rationale: The primary goal of reperfusion therapy in STEMI is to restore blood flow to the heart muscle and minimize damage.
A nurse is administering heparin to a patient with NSTEMI. What is the primary purpose of administering heparin in this scenario?
a. To reduce the heart rate
b. To prevent microemboli from forming and causing further chest pain
c. To improve oxygen delivery to the heart muscle
d. To reduce fluid retention in the lungs
b. To prevent microemboli from forming and causing further chest pain
Rationale: Heparin is used to prevent the formation of microemboli, which can worsen chest pain and contribute to further ischemia in NSTEMI patients.
After a patient with STEMI receives thrombolytic therapy, which of the following should the nurse monitor for to assess the effectiveness of the therapy?
a. Resolution of chest pain and return of the ST segment to baseline
b. Decreased levels of creatinine
c. Increased levels of cardiac biomarkers
d. Increase in blood pressure
a. Resolution of chest pain and return of the ST segment to baseline
Rationale: The most reliable indicators of reperfusion after thrombolytic therapy are the resolution of chest pain and the return of the ST segment to baseline on ECG.
A patient with unstable angina (UA) is being considered for cardiac catheterization. When should this procedure be performed?
a. Immediately after the patient is stabilized or if angina returns or increases in severity
b. Once the patient is discharged from the hospital
c. Only if the patient has a previous history of myocardial infarction
d. Within 6 hours of admission regardless of symptoms
a. Immediately after the patient is stabilized or if angina returns or increases in severity
Rationale: Cardiac catheterization is performed after the patient is stabilized or if angina increases in severity, which helps guide further treatment decisions.
A nurse is caring for a patient who has just been started on dual antiplatelet therapy (DAPT) with aspirin and ticagrelor. The patient is being treated for which condition?
a. STEMI or NSTEMI
b. Stable angina
c. Aortic stenosis
d. Heart failure
a. STEMI or NSTEMI
Rationale: DAPT, consisting of aspirin and ticagrelor, is used to treat patients with STEMI or NSTEMI to reduce the risk of further clot formation.
A nurse is preparing a patient with STEMI for emergent percutaneous coronary intervention (PCI). What is the advantage of PCI over coronary artery bypass graft (CABG) surgery for STEMI patients?
a. PCI is less invasive and associated with a faster recovery time
b. PCI improves long-term survival more than CABG
c. PCI is less effective in preserving heart muscle than CABG
d. PCI does not require local anesthesia
a. PCI is less invasive and associated with a faster recovery time
Rationale: PCI is less invasive than CABG and typically allows for faster recovery, making it the preferred option for STEMI patients.
A nurse is caring for a patient with STEMI who is receiving thrombolytic therapy. What should the nurse closely monitor for after initiating this therapy?
a. Elevated white blood cell count
b. Development of hypotension and signs of bleeding
c. Increased serum glucose levels
d. Increased potassium levels
b. Development of hypotension and signs of bleeding
Rationale: Thrombolytic therapy can cause bleeding, so the nurse should closely monitor for hypotension and signs of bleeding (e.g., bruising, petechiae).
A nurse is providing education to a patient with unstable angina who will be receiving dual antiplatelet therapy. Which statement by the patient indicates understanding of the therapy?
a. “I will stop taking the aspirin if I start to feel better.”
b. “This medication combination will help prevent the formation of blood clots.”
c. “These medications will lower my blood pressure.”
d. “The medications will improve my heart rate.”
b. “This medication combination will help prevent the formation of blood clots.”
Rationale: DAPT helps prevent blood clot formation, which reduces the risk of further ischemia and complications in patients with unstable angina.
The nurse is monitoring a patient with acute coronary syndrome (ACS) who is on bed rest. Which intervention should be implemented to minimize complications associated with prolonged bed rest?
a. Gradually increase activity as tolerated once the patient is stable
b. Encourage the patient to remain on strict bed rest for 72 hours
c. Administer sedatives to prevent anxiety
d. Restrict fluid intake to reduce the risk of pulmonary congestion
a. Gradually increase activity as tolerated once the patient is stable
Rationale: Gradually increasing activity helps prevent complications from prolonged bed rest, such as muscle atrophy and blood clots, while still prioritizing rest and recovery.
A nurse is administering nitroglycerin (NTG) and morphine to a patient with chest pain. Which of the following is the most important aspect of care when using these interventions?
a. Monitor the patient for signs of bradycardia
b. Discontinue NTG and morphine once pain relief is achieved
c. Only administer NTG if the patient is not receiving morphine
d. Evaluate the effectiveness of the interventions regularly
d. Evaluate the effectiveness of the interventions regularly
Rationale: Regular evaluation of the effectiveness of NTG and morphine is crucial to ensure pain relief and to assess the ongoing need for these interventions, as chest pain relief may not indicate the absence of heart disease.
After administering nitroglycerin (NTG) and morphine for chest pain, the nurse notices that the pain has subsided. What should the nurse emphasize to the patient at this point?
a. “The absence of pain means that the heart disease has been cured.”
b. “Pain relief means you can stop your medication regimen.”
c. “Pain relief is important, but continued therapy is necessary to limit myocardial damage.”
d. “You will not need further monitoring if you feel no pain.”
c. “Pain relief is important, but continued therapy is necessary to limit myocardial damage.”
Rationale: The absence of pain does not mean that heart disease is resolved. It is important for the patient to understand that continued therapy is needed to prevent further myocardial damage.
A patient with chest pain receives morphine and nitroglycerin. Which of the following is a priority action after the interventions have been administered and pain is relieved?
a. Decrease the frequency of monitoring vital signs
b. Provide reassurance that the heart disease has been cured
c. Continue to monitor the patient and assess the effectiveness of interventions
d. Discontinue all medications and prepare for discharge
c. Continue to monitor the patient and assess the effectiveness of interventions
Rationale: Continuous monitoring and reassessment of the effectiveness of interventions are critical to ensure that pain relief is maintained and to detect any changes that may require further treatment.
A patient who has recently had a myocardial infarction (MI) develops sustained ventricular tachycardia (VT). What is the nurse’s priority action?
a. Monitor the patient’s blood pressure and heart rate
b. Administer IV fluids to stabilize the patient
c. Notify the healthcare provider for further orders
d. Treat the VT immediately as it is a life-threatening dysrhythmia
d. Treat the VT immediately as it is a life-threatening dysrhythmia
Rationale: Sustained VT is a life-threatening dysrhythmia that requires immediate treatment to prevent further complications such as ventricular fibrillation (VF), which can be fatal.
During continuous ECG monitoring after a myocardial infarction (MI), a nurse notices premature ventricular contractions (PVCs) that are isolated. What is the appropriate nursing intervention?
a. Administer antiarrhythmic medications immediately
b. Perform immediate defibrillation to restore normal rhythm
c. Continue monitoring, as isolated PVCs do not typically require treatment
d. Notify the healthcare provider to start aggressive treatment
c. Continue monitoring, as isolated PVCs do not typically require treatment
Rationale: Isolated PVCs are often not treated unless they become symptomatic or lead to more serious dysrhythmias. Continuous monitoring is appropriate in this situation.
A patient who has had a myocardial infarction (MI) is being monitored for changes in their ECG. Which of the following is the nurse’s most important action when assessing the ST segment?
a. Look for a significant change in the P wave
b. Check for shifts above or below the baseline of the ECG
c. Measure the QRS duration for abnormalities
d. Monitor heart rate for tachycardia
b. Check for shifts above or below the baseline of the ECG
Rationale: Shifts in the ST segment above or below the baseline can indicate reinfarction or ongoing ischemia and should be assessed frequently.
A nurse is monitoring a patient post-MI and notices early signs of heart failure (HF), including crackles and dyspnea. What is the nurse’s priority action?
a. Notify the healthcare provider and prepare for possible intubation
b. Continue monitoring the patient without intervention
c. Increase the patient’s fluid intake to prevent dehydration
d. Assess intake and output and notify the healthcare provider
d. Assess intake and output and notify the healthcare provider
Rationale: Early signs of heart failure, such as crackles and dyspnea, require careful monitoring of intake and output, with appropriate notification to the healthcare provider for further intervention.
The nurse is monitoring a patient on a nasal cannula after a myocardial infarction (MI). What should the nurse do to prevent discomfort related to the use of the nasal cannula?
a. Check the nares for irritation or dryness and adjust the cannula as needed
b. Increase the flow rate of oxygen to improve comfort
c. Switch the patient to a face mask immediately
d. Remove the nasal cannula and provide oxygen by mouth
a. Check the nares for irritation or dryness and adjust the cannula as needed
Rationale: Nasal cannulas can cause irritation or dryness in the nares, which can lead to discomfort. The nurse should check for these issues and make necessary adjustments.
When performing a physical assessment on a patient post-MI, the nurse detects an S3 heart sound. What should the nurse consider as the most likely cause of this finding?
a. Hypertension
b. Tachycardia
c. Pulmonary embolism
d. Fluid overload
d. Fluid overload
Rationale: An S3 heart sound is often associated with fluid overload or early heart failure, which can occur after an MI as the heart struggles to pump effectively.
The nurse is monitoring a patient post-MI for signs of dysrhythmias. Which of the following is considered a life-threatening dysrhythmia most commonly occurring during the initial period after MI?
a. Sinus tachycardia
b. Atrial fibrillation
c. Sustained ventricular tachycardia (VT)
d. Premature atrial contractions (PACs)
c. Sustained ventricular tachycardia (VT)
Rationale: Sustained VT is a life-threatening dysrhythmia that commonly occurs in the initial period after an MI, requiring rapid treatment to prevent further complications.
A patient with an uncomplicated STEMI is being monitored after a PCI procedure. What is the appropriate nursing intervention regarding activity?
a. Allow the patient to sit in a chair within a few hours after the event
b. Maintain complete bed rest for 24 hours to prevent complications
c. Limit all activity for the first 72 hours post-procedure
d. Only allow standing for brief periods to prevent overexertion
a. Allow the patient to sit in a chair within a few hours after the event
Rationale: For an uncomplicated STEMI, the patient can sit in a chair within a few hours after the event, especially if the radial artery was used for PCI. Gradual activity increase is encouraged for early mobilization.
A nurse is caring for a patient with a large STEMI. What is the most likely order regarding activity during the initial period after the event?
a. The patient should be encouraged to walk around the room immediately
b. The patient should slowly increase activity as tolerated, starting with sitting in a chair
c. The patient should only be allowed to use the bathroom and bedpan
d. The patient should remain in bed for an extended period to prevent complications
d. The patient should remain in bed for an extended period to prevent complications
Rationale: A patient with a large STEMI may require a period of bed rest to prevent complications and allow the heart to stabilize before increasing activity levels.
A nurse is helping a patient with UA who has had their angina resolved. The patient asks when they can sit in a chair. What is the nurse’s best response?
a. “You should remain in bed until you are discharged.”
b. “You may sit in a chair now since your angina is resolved and there are no complications.”
c. “You should wait for the doctor’s order before sitting up.”
d. “You need to remain in bed for at least 24 hours after the procedure.”
b. “You may sit in a chair now since your angina is resolved and there are no complications.”
Rationale: For patients with UA and no complications, once the angina is resolved, they can sit in a chair. Early mobilization is encouraged when stable.
A nurse is preparing a discharge plan for a patient recovering from an MI. What is the most important teaching point regarding activity?
a. “You should refrain from all physical activity for the first month after discharge.”
b. “Gradually increase your activity level with more demanding physical tasks.”
c. “You should remain on complete bed rest for the next 6 weeks to ensure full recovery.”
d. “Once you feel no pain, you can resume normal activity without restrictions.”
b. “Gradually increase your activity level with more demanding physical tasks.”
Rationale: The patient should gradually increase their activity level with more demanding tasks to avoid overexertion and promote cardiac rehabilitation. This allows for better recovery and prevents complications.
A nurse is educating a patient recovering from an acute coronary syndrome (ACS). Which of the following is the most important consideration for effective patient teaching during this phase of hospitalization?
a. Teaching complex medical terminology to ensure understanding
b. Providing information about the disease as soon as possible, even if the patient is in crisis
c. Ensuring that patient teaching is brief and provided in simple, clear language
d. Giving the patient all available educational materials at once
c. Ensuring that patient teaching is brief and provided in simple, clear language
Rationale: During a crisis, patients may not be ready to learn new information, so providing teaching in simple and brief terms is key for understanding. Repetition may also be necessary.
A patient is in the ED with chest tightness and asks why they feel this way when climbing stairs. How should the nurse explain the cause of this symptom to the patient?
a. “Your heart’s blood vessels are blocked, reducing oxygen to the muscle.”
b. “Your heart is working harder than usual, causing tightness in your chest.”
c. “Climbing stairs causes your heart to pump too quickly, resulting in tightness.”
d. “The chest tightness is just a temporary symptom of physical activity.”
a. “Your heart’s blood vessels are blocked, reducing oxygen to the muscle.”
Rationale: Chest tightness (angina) occurs when fat and cholesterol block blood vessels supplying oxygen to the heart muscle, reducing its ability to function properly.
A patient with a history of smoking, obesity, and lack of exercise has recently been diagnosed with ACS. He decides to cut back on smoking and start a weight reduction plan but refuses to quit smoking altogether. What is the nurse’s role in response to the patient’s decision?
a. Respect the patient’s decision, but emphasize the importance of quitting smoking for better outcomes
b. Inform the patient that smoking must be completely stopped for recovery
c. Suggest that the patient focus only on weight loss and ignore smoking
d. Discourage any changes in lifestyle, as the patient’s decision may not improve outcomes
a. Respect the patient’s decision, but emphasize the importance of quitting smoking for better outcomes
Rationale: While respecting the patient’s choices, it is important to emphasize the health benefits of quitting smoking, as tobacco use remains a significant risk factor for ACS.
A nurse is preparing a patient for home care after an acute coronary syndrome event. What is the most important aspect of anticipatory guidance for this patient?
a. Providing detailed explanations of all possible complications of ACS
b. Preparing the patient for what to expect during recovery and rehabilitation
c. Giving the patient an extensive list of medications to be taken at home
d. Ensuring that the patient understands the potential need for surgery
b. Preparing the patient for what to expect during recovery and rehabilitation
Rationale: Anticipatory guidance helps the patient understand what to expect during recovery, empowering them to make informed decisions and gain a sense of control.
A nurse is teaching a patient with unstable angina (UA) about chest tightness and physical exertion. What is the best explanation for why the patient feels chest tightness when climbing stairs?
a. “Your heart muscle isn’t strong enough to handle physical activity.”
b. “The blood vessels in your lungs are constricting, causing chest tightness.”
c. “Your chest muscles are overworked, causing tightness.”
d. “The oxygen supply to your heart muscle is reduced due to blocked blood vessels.”
d. “The oxygen supply to your heart muscle is reduced due to blocked blood vessels.”
Rationale: Chest tightness during exertion occurs when blood vessels supplying oxygen to the heart muscle are blocked, resulting in insufficient oxygen for the muscle to function properly.
A patient with acute coronary syndrome is in crisis and unable to absorb much new information. What is the best approach for teaching this patient?
a. Provide written educational materials for the patient to read later
b. Explain everything in detail to ensure comprehensive understanding
c. Limit information to brief, simple explanations and answer questions as needed
d. Delay patient teaching until after the patient has had a full night’s rest
c. Limit information to brief, simple explanations and answer questions as needed
Rationale: During a crisis, patients may not be ready to learn in-depth information. The nurse should provide simple explanations and allow time for questions to ensure the patient understands.
When discussing lifestyle changes with a patient diagnosed with ACS, the nurse should focus on which of the following?
a. Encouraging the patient to quit smoking and lose weight as a priority
b. Emphasizing the need to exercise intensely every day to improve cardiovascular health
c. Advising the patient to avoid all physical activity until complete recovery
d. Suggesting that the patient focus only on controlling cholesterol levels
a. Encouraging the patient to quit smoking and lose weight as a priority
Rationale: Smoking cessation and weight loss are two key risk factor modifications for improving the patient’s health and reducing the likelihood of further ACS events.
A nurse is preparing to educate a patient recovering from ACS about the importance of activity during recovery. The nurse should emphasize that the patient should:
a. Avoid any physical activity for the first two weeks to promote full recovery
b. Gradually increase activity to prevent overexertion and support recovery
c. Only perform light activity until all medications are stopped
d. Engage in daily strenuous exercise to improve heart health quickly
b. Gradually increase activity to prevent overexertion and support recovery
Rationale: Gradual increase in activity is essential to allow the patient’s heart to adjust and avoid overexertion, promoting optimal recovery and rehabilitation.
A nurse is teaching a patient with ACS about the importance of managing risk factors. The patient says, “I’ll work on losing weight and getting more exercise, but I just can’t quit smoking.” What should the nurse do next?
a. Discourage the patient from making any lifestyle changes until they are ready to quit smoking
b. Agree with the patient and suggest focusing only on weight loss and exercise
c. Respect the patient’s decision but continue to emphasize the importance of smoking cessation
d. Advise the patient to ignore the smoking habit, as it’s not as important as weight loss
c. Respect the patient’s decision but continue to emphasize the importance of smoking cessation
Rationale: The nurse should respect the patient’s decision but continue to emphasize the importance of quitting smoking for long-term health benefits and to reduce future ACS risks.
A nurse is evaluating the outcomes for a patient recovering from acute coronary syndrome (ACS). Which of the following is an expected outcome for this patient?
a. The patient will maintain a high level of physical activity with no restrictions
b. The patient will describe the disease process and measures to reduce risk factors
c. The patient will report constant shortness of breath due to the illness
d. The patient will experience increased anxiety and decreased sense of self-control
b. The patient will describe the disease process and measures to reduce risk factors
Rationale: An expected outcome for the ACS patient is to be able to describe the disease process and identify measures to reduce risk factors, indicating understanding of their condition and management.
A hospitalized patient with a history of chronic stable angina tells the nurse they are having chest pain. Which information about ischemia would the nurse use as a basis for planning care?
a. It will always progress to myocardial infarction.
b. It can be relieved by rest, nitroglycerin, or both.
c. It is often associated with vomiting and extreme fatigue.
d. It indicates that irreversible myocardial damage is occurring.
b. It can be relieved by rest, nitroglycerin, or both.
he nurse is caring for a patient who is 2 days post MI. The patient reports that chest pain when taking a deep breath. Which action would be a priority?
a. Notify the provider STAT and obtain a 12-lead ECG.
b. Obtain vital signs and auscultate for a pericardial friction rub.
c. Apply high-flow O2 by face mask and auscultate breath sounds.
d. Medicate the patient with an opiate analgesic and reevaluate in 30 minutes.
b. Obtain vital signs and auscultate for a pericardial friction rub.
A patient is in the ICU with a diagnosis of NSTEMI. Which drugs would the nurse expect the patient to receive? (select all that apply)
a. Oral statin therapy
b. Antiplatelet therapy
c. Thrombolytic therapy
d. Prophylactic antibiotics
e. Intravenous nitroglycerin
a. Oral statin therapy
b. Antiplatelet therapy
e. Intravenous nitroglycerin
A patient who has had chest pain for several hours is admitted with a diagnosis of rule out acute myocardial infarction (AMI). Which laboratory test is most specific for the nurse to monitor in determining whether the patient has had an AMI?
a. Myoglobin
b. Homocysteine
c. C-reactive protein
d. Cardiac-specific troponin
d. Cardiac-specific troponin
Rationale: Troponin levels increase about 4 to 6 hours after the onset of myocardial infarction (MI) and are highly specific indicators for MI. Myoglobin is released within 2 hours of MI, but it lacks specificity and its use is limited. The other laboratory data are useful in determining the patient‘s risk for developing coronary artery disease but are not helpful in determining whether an acute MI is in progress.
Heparin is ordered for a patient with a non–ST-segment-elevation myocardial infarction (NSTEMI). How should the nurse explain the purpose of the heparin to the patient?
a. “Heparin enhances platelet aggregation at the plaque site.”
b. “Heparin decreases the size of the coronary artery plaque.”
c. “Heparin prevents the development of new clots in the coronary arteries.”
d. “Heparin dissolves clots that are blocking blood flow in the coronary arteries.”
c. “Heparin prevents the development of new clots in the coronary arteries.”
Rationale: Heparin helps prevent the conversion of fibrinogen to fibrin and decreases coronary artery thrombosis. It does not change coronary artery plaque, dissolve already formed clots, or enhance platelet aggregation.
A patient is admitted to the emergency department and diagnosed as having an ST-segment-elevation myocardial infarction (STEMI). Which question would the nurse ask to determine whether the patient is a candidate for thrombolytic therapy?
a. “Do you have any allergies?”
b. “Did you take aspirin today?”
c. “What time did your pain begin?”
d. “Can you rate the pain on a 0 to 10 scale?”
c. “What time did your pain begin?”
Rationale: Thrombolytic therapy criteria include chest pain for less than 12 hours with 12-lead ECG findings consistent with an acute STEMI. The other information is not a factor in the decision about thrombolytic therapy.
The nurse is administering a thrombolytic agent to a patient having an acute myocardial infarction. Which patient data indicates that the nurse should stop the drug infusion?
a. Bleeding from the gums
b. An increase in blood pressure
c. Decreased level of consciousness
d. A nonsustained episode of ventricular tachycardia
c. Decreased level of consciousness
Rationale: The change in level of consciousness indicates that the patient may be experiencing intracranial bleeding, a possible complication of thrombolytic therapy. Some bleeding of the gums is an expected side effect of the therapy but not an indication to stop infusion of the thrombolytic medication. A decrease in blood pressure could indicate internal bleeding. A nonsustained episode of ventricular tachycardia is a common reperfusion dysrhythmia and may indicate that the therapy is effective.
After having a myocardial infarction (MI) and successful percutaneous coronary intervention, the patient states, “It was just a little chest pain. As soon as I get out of here, I‘m going for my vacation as planned.” Which reply would be most appropriate for the nurse to make?
a. “What do you think caused your chest pain?”
b. “Where are you planning to go for your vacation?”
c. “Sometimes plans need to change after a heart attack.”
d. “Recovery from a heart attack takes at least a few weeks.”
a. “What do you think caused your chest pain?”
Rationale: When the patient is experiencing denial, the nurse should assist the patient in testing reality until the patient has progressed beyond this step of the emotional adjustment to MI. Asking the patient about vacation plans reinforces the patient‘s plan, which is not appropriate in the immediate post-MI period. Reminding the patient in denial about the MI is likely to make the patient angry and lead to distrust of the nursing staff.
A patient who is being admitted to the emergency department with intermittent chest pain gives the following list of daily medications to the nurse. Which medication has the most immediate implications for the patient‘s care?
a. Sildenafil (Viagra)
b. Furosemide (Lasix)
c. Warfarin (Coumadin)
d. Diltiazem (Cardizem)
a. Sildenafil (Viagra)
Rationale: The nurse will need to avoid giving nitrates to the patient because nitrate administration is contraindicated in patients who are using sildenafil because of the risk of severe hypotension caused by vasodilation. The other home medications should be documented and reported to the health care provider but do not have as immediate an impact on decisions about the patient‘s treatment.
Which assessment finding in a patient who has had coronary artery bypass grafting using a right radial artery graft is most important for the nurse to communicate to the health care provider?
a. Complaints of incisional chest pain
b. Pallor and weakness of the right hand
c. Fine crackles heard at both lung bases
d. Redness on both sides of the sternal incision
b. Pallor and weakness of the right hand
Rationale: The changes in the right hand indicate compromised blood flow, which requires immediate evaluation and actions, such as prescribed calcium channel blockers or surgery. The other changes are expected or require nursing interventions.
A patient with acute coronary syndrome has returned to the coronary care unit after having angioplasty with stent placement. Which assessment data indicate the need for immediate action by the nurse?
a. Report of chest pain
b. Heart rate 102 beats/min
c. Pedal pulses 1+ bilaterally
d. Blood pressure 103/54 mm Hg
a. Report of chest pain
Rationale: The patient‘s chest pain indicates that restenosis of the coronary artery may be occurring and requires immediate actions, such as administration of oxygen and nitroglycerin, by the nurse. The other information indicates a need for ongoing assessments by the nurse.
A patient admitted to the coronary care unit (CCU) with an ST-segment-elevation myocardial infarction (STEMI) is restless and anxious. The blood pressure is 86/40 mm Hg, and heart rate is 132 beats/min. Based on this information, which patient problem is the priority?
a. Acute pain
b. Deficient knowledge
c. Impaired cardiac function
d. Health maintenance alteration
c. Impaired cardiac function
Rationale: The hypotension and tachycardia indicate decreased cardiac output and shock from the impaired function of the damaged myocardium. This will result in decreased perfusion to all vital organs (e.g., brain, kidney, heart) and is a priority.
When admitting a patient with a non–ST-segment-elevation myocardial infarction (NSTEMI) to the intensive care unit, which action would the nurse perform first?
a. Attach the heart monitor.
b. Obtain the blood pressure.
c. Assess the peripheral pulses.
d. Auscultate the breath sounds.
a. Attach the heart monitor.
Rationale: Because dysrhythmias are the most common complication of myocardial infarction (MI), the first action should be to place the patient on a heart monitor. The other actions are also important and should be accomplished as quickly as possible.
Which information about a patient receiving thrombolytic therapy for an acute myocardial infarction is most important for the nurse to communicate to the health care provider?
a. An increase in troponin levels from baseline
b. A large bruise at the patient‘s IV insertion site
c. No change in the patient‘s reported level of chest pain
d. A decrease in ST-segment elevation on the electrocardiogram
c. No change in the patient‘s reported level of chest pain
Rationale: Continued chest pain suggests that the thrombolytic therapy is not effective and that other interventions such as percutaneous coronary intervention may be needed. Bruising is a possible side effect of thrombolytic therapy, but it is not an indication that therapy should be discontinued. The decrease of the ST-segment elevation indicates that thrombolysis is occurring, and perfusion is returning to the injured myocardium. An increase in troponin levels is expected with reperfusion and is related to the washout of cardiac biomarkers into the circulation as the blocked vessel is opened.
The nurse obtains the following data when assessing a patient who experienced an ST-segment-elevation myocardial infarction (STEMI) 2 days previously. Which information is most important to report to the health care provider?
a. The troponin level is elevated.
b. The patient denies having a heart attack.
c. Bilateral crackles in the mid-lower lobes.
d. Occasional premature atrial contractions (PACs).
c. Bilateral crackles in the mid-lower lobes.
Rationale: The crackles indicate that the patient may be developing heart failure, a possible complication of myocardial infarction (MI). The health care provider may need to order medications such as diuretics or angiotensin-converting enzyme inhibitors for the patient. Elevation in troponin level at this time is expected. PACs are not life-threatening dysrhythmias. Denial is a common response in the immediate period after the MI.